EMT Test 6 Part 2

Ace your homework & exams now with Quizwiz!

Neglect

Condition when the patient forgets about the injured side after a stroke.

pulmonary edema

Pink , frothy sputum is most likely from

Management of a patient with ACS

1. Place the patient in position of comfort (Fowler) 2. Administering high O2 via Non-reb mask 3. Assist the patient of nitro if medical direction authorizes

What are the 2 most common sites for an aneurysm to occur?

1) aorta 2) brain

Shockable rhythms

1. V-Fib 2. Pulseless ventricular tachycardia 3. Ventricular tachycardia

The spinal cord exits the cranium through the: A. foramen magnum. B. vertebral foramen. C. foramen lamina. D. cauda equina.

A. foramen magnum.

Signs and symptoms of a hypertensive emergency would MOST likely be delayed in patients A. have chronic hypertension. B. regularly take illegal drugs. C. are older than 40 years of age. D. have had a stroke in the past.

A. have chronic hypertension.

what most accurately describes a simple partial seizure?

: A seizure that begins in one extremity

Which of the following is NOT a typical indication of congestive heart failure​ (CHF)? A. Low blood pressure B. ​Wet-sounding breath sounds C. Increased heart rate D. Productive cough

A. Low blood pressure

Which of the following is NOT a reason that an AED may indicate that there is "no shock advised"?

The patient is in ventricular fibrillation or pulseless ventricular tachycardia.

Which of the following statements regarding the administration of nitroglycerin tablets is true?

The patient may complain of a headache following administration.

Postictal state

The period following a seizure that lasts 5 to 30 minutes; characterized by labored respirations and some degree of altered mental status.

SA node

The primary pacemaker of the heart

popliteal

The pulse you feel behind the knee

Stroke

An interruption of blood flow to the brain that results in the loss of brain function; also called a cerebrovascular accident (CVA).

Risk factors for AMI that cannot be controlled include:

family history.

The iliac arteries immediately subdivide into the

femoral arteries.

Blood that is ejected from the right ventricle

flows into the pulmonary arteries.

Cerebrovascular accident (CVA)

An interruption of blood flow to the brain that results in the loss of brain function; also called a stroke.

What is the function of the left atrium?

It receives oxygenated blood from the lungs.

What does ischemia mean?

Lack of oxygen due to inadequate blood supply due to blockage of blood vessels leading to that area

Treatment of a migraine

- High flow oxygen - Provide a darkened and quiet environment - Do not use lights and sirens during transport

What is an aneurysm?

A weak spot of an artery wall that begins to dilate(balloon)

aortic aneurysm

A weakness in the wall of the aorta that makes it susceptible to rupture.

Which of the following MOST accurately describes a simple partial seizure?

A seizure that begins in one extremity.

Aura

A sensation experienced before a seizure; serves as a warning sign that a seizure is about to occur.

Simple Partial Seizure VS Complex Partial Seizure

A simple partial seizure may cause twitching in the muscles of the extremities & may move slowly from 1 body part 2 another. In a complex partial seizure a patient has an AMS & does not interact normally w their environment. Complex partial seizures result from abnormal discharge from the temporal lobe of the brain.

Coma

A state of profound unconsciousness from which the patient cannot be roused.

COMA

A state of profound unconsciousness which 1 cannot B roused.

Thrombus

A stationary blood clot along the wall of the blood vessel a clot formed of blood and plaque ATTACHED to the inner wall of an artery or vein

Aneurysm

A swelling or enlargement of the wall of a blood vessel that results from weakening of the vessel wall.

Delirium

A temporary change in mental status characterized by disorganized thoughts, inattention, memory loss, disorientation, striking changes in personality and affect, hallucinations, delusions, or a decreased level of consciousness.

acute coronary syndrome

A term used to describe a group of symptoms caused by myocardial ischemia; include angina, and AMI

Hemorrhagic stroke

A type of stroke that occurs as a result of bleeding inside the brain.

Ischemic stroke

A type of stroke that occurs when blood flow to a particular part of the brain is cut off by a blockage (eg, a blood clot) inside a blood vessel.

What is the most important factor in assessment of the ability of the cardiovascular system to meet the body's metabolic demands?

Adequacy of perfusion

What is NOT a role of the EMT in providing emergency cardiac​ care?

Administration of epinephrine

Which of the following is NOT a role of the EMT in providing emergency cardiac care?

Administration of epinephrine

THROMBOSIS

Clotting of the cerebral arteries that may result in the interruption of cerebral blood flow & subsequent stroke.

EMBOLUS

Clotting that forms in a remote area & travels 2 the site of blockage.

You arrive on-scene of a 60-year-old female patient in cardiac arrest. What is the best airway that you, as an EMT, can provide for the patient?

Combitube

You are examining a patient whose signs include lip smacking, jerking of the left arm, and agitation. Based on these signs, what type of seizure is this patient experiencing?

Complex partial

What is coronary artery disease?

Conditions that narrow or block the arteries of the heart

Which of the following MOST accurately describes what the patient will experience during the postictal state that follows a seizure?

Confusion and fatigue.

Common causes of Epileptic seizures

Congenital origin

What is CHF?

Congestive Heart Failure, a condition of excessive fluid buildup in the lungs and or other organs and body parts because of the inadequate pumping of the heart

What effect does spontaneous deep inspiration have on venous return?

Increased venous return

Which of the following statements regarding nitroglycerin is correct?

Nitroglycerin usually relieves anginal chest pain within 5 minutes.

What does "apnea" mean?

No breathing

Simple partial seizure

No change occurs in the patient's level of consciousness.

Which of the following people is al lowed to continue providing patient care when the patient is to be "cleared" for delivery of a shock via the defibrillator?

None of the above

You arrive on-scene of a 56-year-old female patient in cardiac arrest. What is the first airway you should administer?

Oropharyngeal airway

You are treating a patient with signs and symptoms of a myocardial infarction (MI). What is the most important drug you should administer?

Oxygen

Berry aneurysm

This type of aneurysm resembles a tiny balloon (or berry) that juts out from the artery.

Which of the following is a contraindication for​ aspirin? A. Liver disease B. Cardiovascular disease C. Having taken nitroglycerin D. Asthma

asthma

Which of the following would NOT be corrected by​ defibrillation? A. Course ventricular fibrillation B. Ventricular tachycardia C. Fine ventricular fibrillation D. Asystole

asystole

Administer oxygen to your chest pain patient to maintain an oxygen saturation level​ of: A. up to 94 percent. B. at least 94 percent. C. up to 100 percent. D. at least 90 percent.

at least 94 percent.

Narrowing of the coronary arteries due to a buildup of fatty deposits is called:

atherosclerosis.

The ability of cardiac muscle cells to contract spontaneously without a stimulus from a nerve source is called:

automaticity.

The electrical stimulus that originates in the heart's primary pacemaker is controlled by impulses from the brain that arrive by way of the:

autonomic nervous system.

You and your partner arrive at the scene of a middle-aged man who collapsed about 5 minutes ago. He is unresponsive, apneic, and pulseless. Bystanders are present, but have not provided any care. You should:

begin high-quality CPR and apply the AED without delay.

After 3 shocks of the AED the EMT should

begin to transport the patient with high quality CPR

What is the MOST common reason for the​ AED's delivering a shock​ inappropriately? A. Mechanical error B. Software error C. Electrical interference D. User error

user error

What are the primary effects of aspirin?

Reduces blood clotting

The resistance to ventricular ejection also is known as?

Afterload

aneurysm?

a dilation, or ballooning, of a weekend section of the wall of an artery.

Dysrhythmia

a disturbance in heart rate and rhythm

What are the primary effets of nitroglycerin?

Relaxes blood vessels Decreases workload of heart

AED automated external defib.

Releases a shock when in cardiac arrest.

Edema

Swelling resulting from a build-up of fluid in the tissues

APHASIA

The inability 2 understand / produce speech

Aphasia

The inability to understand and/or produce speech.

Dura

The leathery covering of the brain that lies next to the skull.

Aorta

The main artery that supplies the body with oxygenated blood, comes from the left ventricle.

Syncope

A fainting spell or transient loss of consciousness.

When caring for a patient documented hypoglycemia, you should be MOST alert for:

A seizure.

What is a dysrhythmia?

An irregular, or absent heart rhythm.

Acute coronary syndrome (ACS)

Any condition brought on by a sudden reduction or blockage of blood flow to the heart.

Subdural bleeding

Bleeding beneath the dura but outside the brain.

Sudden death

Cardiac arrest occurring with 2 hours of onset symptoms.

ischemia

Lack of oxygen to tissues

Hypoperfusion

Shock

HEMIPARESIS

Weakness on 1 side of the body

occlusion?

a blockage, as of an artery, by fatty deposits.

The Cardiovascular System is composed of...

heart and blood vessels

The descending aorta divides into the two iliac arteries at the level of the

umbilicus.

Pulse-less Electrical Activity (PEA)

A condition in which the heart has an electrical rhythm but is not generating a pulse

nitroglycerins trade name?

nitrostat

POSTICTAL STATE

A period following a seizure that lasts between 5 & 30 min;characterized by labored respirations & some degree of altered mental status.

HEMORRHAGIC STROKE

1 of the 2 main types of stoke;occurs as a result of bleeding inside the brain.

What is an occlusion?

A blockage, caused by a thrombus that grows to a certain size usually

What is sudden death as it related to heart attacks?

A cardiac arrest that occurs within two hours of the onset of symptoms

GLASGOW COMA SCALE

All patients w AMS should have a Glasgow Coma score calculated.

sympathetic

Fight or flight system

What is angina pectoris?

Literally, a pain in the chest

Apnea

No breathing

DYSARTHRIA

Slurred Speech

femorals

The arteries in the groin area

Dilution

Thinning or weakening

acute myocardial infarction

A heart attack is known as an (full words ,no acronyms )

Downers

Depressants

syncope

Temporary LOC, fainting

systolic

The pressure exerted upon the cell walls when the heart is contracting.

AURA

A sensation experienced prior 2 a seizure;serves as a warning sign that a seizure is about 2 occur.

Non-shockable rhythms

1. Aystole 2. PEA

Balloon angioplasty

A catheter inserted and can be inflated like a balloon to reopen circulation to the heart.

Parasympathetic

Rest and digest system

Pedal Edema

accumulation of fluid in the feet or ankles

The posterior tibial pulse can be palpated

behind the medial malleolus.

Apnea

no breathing

Hypotensive

systolic < 90 mmHg

Tachycardia

HR above 100 BPM in adults. When the heart rate is fast.

Back part of the cerebrum

Processes sight.

perfusion

The flow of blood and nutrients to the tissues.

Hallucinogens

Mind altering drugs

Deoxygenated blood from the body returns to the

right atrium.

Edema

swelling resulting from a buildup of fluid in the tissues

DO NOT defibrillate a patient if:

- Anyone is touching the patient (Stand clear.) - Patient is wearing a nitroglycerin patch (Remove patch.) - Patient is wet or patient is lying in water (Dry patient.) - Patient is touching metal (Move away from metal.)

Signs and symptoms of Acute Coronary syndrome (ACS):

- Pain, pressure, or discomfort in the chest or upper abdomen (epigastrium) - Difficulty breathing - Palpitations - Sudden onset of sweating and nausea or vomiting - Anxiety (feeling of impending doom, irritability) - Unusual generalized weakness - Abnormal pulse (rapid, slow, or irregular) - Abnormal blood pressure

What are the indications that must be met to give the patient (or help them take) nitroglycerin?

- Patient complains of chest pain - Patient has a history of cardiac problems - Patient's physician has prescribed nitroglycerin (NTG) - Patient has the nitroglycerin with him - Systolic blood pressure meets your protocol criterial (usually greater than 90-100 systolic) - Patient has not taken Viagra or a similar drug for ED within 48-72 hours - Medical direction authorizes administration of the medication

Management of cardiac arrest

- Perform one- and two-rescuer CPR - Use an automated external defibrillator - Request Advanced Life Support (when available) to continue the chain of survival - Use a bag-valve mask device with oxygen - Use a flow-restricted, oxygen-powered ventilation device - Lift and move patients - Suction a patient's airway - Use airway adjuncts (oropharyngeal & nasopharyngeal) - Take standard precautions to protect yourself - Interview bystanders and family members to obtain facts related to the arrest

Signs and symptoms of CHF:

-Tachycardia -Dyspnea -Normal or elevated BP -Cyanosis -Diaphoresis/cool and clammy skin -Pulmonary edema -Sometimes coughing up frothy white/pink sputum -Anxiety or confusion (due to hypoxia) (caused by poor O2/CO2 exchange) -Pedal edema -Engorged, pulsating neck veins (late sign) -Enlarged liver and spleen with abdominal distention (late sign)

ISCHEMIC STROKE

1 of the 2 main types of stroke;occurs when blood flow 2 a particular part of the brain is cut off by blockage (a clot) inside a blood vessel.

nitroglycerin dosage?

1 spray or tablet sublingual every 5 minutes dependent on pain level.

Signs & Symptom of Stroke / Transient Ischemic Attack

1) Confused, Dizzy, Weak 2) Decreasing or increasing LOC 3) Combative or uncooperative or restless 4) Facial drooping, inability 2 swallow, tongue deviation 5) Double vision or blurred vision 6) Difficulty speaking or absence speech 7) Decreased or absent movement of 1 or more extremities 8) Headache 9) Decreased or absent sensation in 1 or more extremities or other areas of the body 10) Coma

Conditions That My Mimic Stroke

1) Hypoglycemia 2) Postictal State 3) Subdural / Epidural Bleeding

What are the five elements that affect the survival of cardiac arrest patients?

1) Immediate recognition and activation 2) Early CPR 3) Rapid defibrillation 4) Effective advanced life support 5) Integrated post-cardiac arrest care

Transport ACS patient immediately IF the patient has any one of the following:

1) No history of cardiac problems 2) History of cardiac problems but does not have nitroglycerin 3) Systolic BP below 90 or 100

What are non-shockable heart rhythms?

1) Pulseless electrical activity (PEA) 2) Asystole

SEIZURE MANAGEMENT

1) Safety of patient / position 2) ABCs, consider nasopharyngeal airway 3) Oxygen / suction 4) Pulse oximetry 5) Emotional support

Management Of Patient W Stroke Assessment Findings

1) Scene safety & PPE 2) ABCs / Position 3) Oxygen / Suction 4) Emotional support 5) Rapid transport

SEIZURE ASSESSMENT

1) Spasms, muscle contractions 2) Bite tongue, increased secretions 3) Sweating 4) Cyanosis 5) Unconscious gradually increasing LOC 6) May cause shaking or tremors & no LOC 7) Incontinent 8) Amnesia of event

What are shockable heart rhythms?

1) Ventricular fibrillation 2) Ventricular tachycardia ( if the patient is unconscious)

Nitroglycerin can be administered under the conditions that..

1. Authorization by medical direction 2. Patients physician prescribed the medication 3. Blood pressure is greater then 100 systolic

Signs and symptoms of Acute Coronary Syndrome

1. Difficulty breathing 2. Sudden onset of sweating with nausea and vomiting 3. Pain in the chest or upper abdomen

Do not defib. if what?

1. Someone is touching patient 2. Patient is wet 3.patient is wearing nitro patch (remove patch) 4. Patient is touch or wearing any metal.

What is the dosage for aspirin given to treat acute coronary​ syndrome? A. 81 mg of chewable baby aspirin B. 162 to 324 mg of chewable baby aspirin C. 40 mg of coated adult aspirin D. 81 mg of coated adult aspirin

162 to 324 mg of chewable baby aspirin

Aspirin dosage?

162-324 (2-4 81 mg tablets)

Approximately how much blood can be lost before circulatory function and pressures are affected?

20% to 25% of total blood volume can be lost without altering circulatory function and pressures

CAUSES OF MIGRAINES

2nd most common cause of headaches. Caused by changes in the blood vessel @ the base of the brain. Described as pounding, throbbing or pulsating. Associated w nausea, vomiting or visual changes. Can last for several days.

Assuming your protocolal lows the administration of nitroglycerin when certain conditions exist, what is the maximum number of tablets to be administered in the prehospital setting?

3

Typically, how many shocks should you administer on scene before​ transport, assuming that the shocks are​ ineffective? A. 3 B. 4 C. 2 D. 1

3

Maximum number of doses of Nitro

3 doses

What is the max dosage for aspirin?

324mg in the form of four 81mg tablets

If a shock is not​ indicated, how many cycles of CPR will you give before analyzing​ again? A. 2 B. 10 C. 15 D. 5

5 2 minutes or 5 cycles of 30:2

Usually angina attacks don't last longer than _________?

5 minutes

At rest, approximately what percentage of total blood volulme is stored in the venous system?

64%

Which of the following patients would most likely have acute coronary syndrome and not complain of chest pain? A A 39-year-old male B A 40-year-old female C An 80-year-old female D A 79-year-old male

80-year-old female - older patients and women primarily do not have the typical presentation of chest discomfort

You are performing a Glasgow Coma Scale on a patient who opens her eyes in response to pain, is confused in her responses to questions, and has abnormal flexion. What is this patient's GCS score and corresponding state of dysfunction?

9 - moderate dysfunction

Brachycardia

< 60 bpm

thrombus?

A clot formed from blood or plaque attached to the inner wall of an artery or vein.

What is a Thrombus?

A clot formed of blood and plaque attached to the inner wall of an artery or vein

Which of the following patients should have an automated external defibrillator applied?

A 19-year-old college athlete who collapsed during football practice and is pulseless and apneic

What is the difference between a stroke and a transient ischemic attack?

A TIA resolves completely within 24 hours of onset.

Embolus

A blood clot or other substance in the circulatory system that travels to a blood vessel where it causes a blockage.

Thrombosis

A blood clot, either in the arterial or venous system. When the clot occurs in a cerebral artery, it may result in the interruption of cerebral blood flow and subsequent stroke.

HYPOGLYCEMIA

A condition characterized by low blood glucose level. Since glucose is also required for brain function, hypoglycemia may mimic conditions associated w stroke such as hemiparesis. THE MAIN DIFFERENCE is a patient w a stroke may B alert & attempting 2 communicate normally whereas a patient w hypoglycemia almost always has an altered or decreased level of consciousness.

STATUS EPILEPTICUS

A condition in which seizures recur every few min. or last more than 30 min.

Status epilepticus

A condition in which seizures recur every few minutes or last longer than 30 minutes.

Ventricular tachycardia

A condition on which the heart beat is quite rapid. A condition in which the heartbeat is quite rapid; if rapid enough, the heart chambers will not be able to fill with enough blood to produce sufficient blood flow

Epilepsy

A disorder in which abnormal electrical discharges occur in the brain, causing seizure and possible loss of consciousness.

Atherosclerosis

A disorder in which calcium and cholesterol build up inside the walls of the blood vessels, forming plaque, potentially leading to a partial or complete blockage of blood flow.

Transient ischemic attack (TIA)

A disorder of the brain in which brain cells temporarily stop functioning because of insufficient oxygen, causing stroke-like symptoms that resolve completely within 24 hours of onset.

TRANSIENT ISCHEMIC ATTACK (TIA)

A disorder of the brain in which brain cells temporarily stop working b/c of insufficient oxygen, causing stroke like Sx that resolve completely within 24 hrs of onset.

Phenytoin

A drug that is used to control seizures, may itself cause seizures if the person takes too much.

A - Alcohol E - Epilepsy, endocrine, electrolytes I - Insulin O - Opiates and other drugs U - Uremia (kidney failure) T - Trauma I - Infection P - Poisoning, psychogenic causes S - Shock, stroke, seizure, syncope, space-occupying lesion, subarachnoid hemorrhage.

A helpful mnemonic to use when reviewing the possible causes of altered mental status.

ISCHEMIA

A lack of oxygen in the cells of the brain that causes them not to function properly.

Ischemia

A lack of oxygen that deprives tissues of necessary nutrients, resulting from partial or complete blockage of blood flow; potentially reversible because permanent injury has not yet occurred.

implanted pacemaker

A lump is present over the left nipple of a pt., this is most likely an

Who is a typical cardiac arrest victim?

A male in his 60's

Nitroglycerin

A medication that diatlates the blood vessels

Seizure

A neurologic episode caused by a surge of electrical activity in the brain; can be a convulsion characterized by a generalized, uncoordinated muscular activity, and can be associated with loss of consciousness.

- Sudden onset of symptoms - Explosive/thunderclap pain - Altered mental status - Age > 50 - Depressed immune system (known to be at higher risk for infection) - Neurologic deficits - Neck stiffness/pain - Fever - Changes in vision - One-sided paralysis or weakness

A patient who has a headache associated with any of the following should be evaluated for a potentially life-threatening condition:

What is a balloon angioplasty or balloon catheterization?

A procedure in which a catheter is inserted with a small balloon that can be inflated to clear a clot and reopen circulation to the heart

Partial (focal) seizure

A seizure affecting a limited portion of the brain.

PARTIAL SEIZURE

A seizure affecting a limited portion of the brain. Partial seizure may involve some twitching of the muscles but it is not characterized by the dramatic twitching seen in a generalized seizure. Partial seizures can B simple or complex

GENERALIZED SEIZURE

A seizure characterized by severe twitching of all of the body's muscles that may last several minutes or more;formerly known as grand mal seizures.

Generalized (tonic-clonic) seizure

A seizure characterized by severe twitching of all the body's muscles that may last several minutes or more; formerly known as a grand mal seizure.

Idiopathic seizure

A seizure for which the cause cannot be determined.

Which of the following will provide a cardiac arrest patient the greatest chance of​ survival? A. Defibrillation within two minutes B. Advanced airway management C. Rapid transport D. Early advanced cardiac life support

A. Defibrillation within two minutes

Which intervention is proven to be the most effective is obtaining a return of spontaneous circulation​ (ROSC) in a cardiac arrest​ patient? A. Early defibrillation B. ​High-concentration oxygen by​ bag-valve mask C. Early CPR D. Combitube

A. Early defibrillation

Which of the following BEST describes the continued need for prehospital advanced cardiac life​ support, even when EMTs in the community carry​ AEDs? A. Not all cardiac arrests are due to problems that respond to defibrillation. B. In cases of refractory or recurrent ventricular​ fibrillation, the use of medications may decrease the chances of defibrillating successfully. C. The patient may need medications to support his cardiac rhythm and blood pressure prior to attempting defibrillation. D. All of the above

A. Not all cardiac arrests are due to problems that respond to defibrillation.

Which of the following statements about the AED is​ correct? A. The AED can encounter difficulty in analyzing a rhythm in a moving vehicle. B. The AED can always analyze the rhythm in a moving vehicle. C. Biphasic AEDs can analyze the rhythm in a moving vehicle. D. AEDs can analyze in a moving vehicle but cannot shock.

A. The AED can encounter difficulty in analyzing a rhythm in a moving vehicle.

After administration of nitroglycerin to a patient with chest​ pain, which is a side effect you MOST might expect to​ see? A. The patient develops a headache. B. The patient becomes dehydrated. C. The patient becomes irritable. D. The patient develops hypertension.

A. The patient develops a headache.

Which of the following statements regarding the administration of nitroglycerin tablets is​ TRUE? A. The patient may complain of a headache following administration. B. An increase in blood pressure should be expected. C. If a​ patient's pulse rate changes following nitroglycerin​ administration, this indicates an allergic reaction. D. It takes 20 to 30 minutes for nitroglycerin to have an effect.

A. The patient may complain of a headache following administration.

Automatic defibrillation is NOT appropriate in most cases of infant cardiac arrest due to which of the​ following? A. Ventricular fibrillation is not the primary cause of cardiac arrest in the pediatric patient. B. The energy delivered by the AED would not be effective on an infant. C. AEDs cannot effectively determine rhythms on patients weighing less than 20 pounds. D. None of the above

A. Ventricular fibrillation is not the primary cause of cardiac arrest in the pediatric patient.

One of the side effects of administering nitroglycerin to a cardiac patient could​ involve: A. a drop in blood pressure. B. hypertension. C. chest pain. D. apnea.

A. a drop in blood pressure.

Your patient was pulseless but had a return of spontaneous pulses after one shock from the AED. Before​ transport, you notice that the patient is again unresponsive. You should​ NEXT: A. check the pulse. B. transport the patient. C. continue ventilations. D. assess the blood pressure.

A. check the pulse.

ALS personnel have just placed an advanced airway in an adult cardiac arrest patient. You​ should: A. deliver asynchronous ventilations at 8 to 10 breaths per minute. B. deliver 2 breaths after every 30 compressions. C. ventilate the patient at 12 to 20 breaths per minute. D. ventilate the patient twice after every 15 compressions.

A. deliver asynchronous ventilations at 8 to 10 breaths per minute.

If a patient experiencing cardiac compromise does not complain of​ dyspnea, then it​ is: A. important to ask the patient about breathing difficulty. B. likely that the patient is not experiencing breathing difficulty. C. probable that the patient is experiencing nausea instead. D. a sign that the patient is not in severe ischemic pain.

A. important to ask the patient about breathing difficulty.

Which of the following is a condition that can result from fatty deposits forming in the inner lining of the​ arteries? A. Coronary artery disease B. Arteriosclerosis C. Aneurysm D. Coronary thrombosis

A. Coronary artery disease

Which of the following signs is commonly observed in patients with right-sided heart failure? A. Dependent edema B. Flat jugular veins C. Pulmonary edema D. Labored breathing

A. Dependent edema

What stimulates a mechanical contraction of the cells of the​ heart, creating a mechanical squeeze used to push​ blood? A. Electrical energy B. Left and right atria C. Right ventricle D. Left ventricle

A. Electrical energy

What is​ (are) the first​ organ(s) perfused by blood leaving the​ heart? A. Heart B. Coronary arteries C. Liver D. Lungs

A. Heart

Which of the following is an expected side effect of​ aspirin? A. Heartburn B. Hypertension C. Increased clotting D. Fever

A. Heartburn

Which of the following is the beneficial action of a beta blocker​ medication? A. Slows the heart rate B. Increases the amount of oxygen needed by the myocardium C. Causes​ vasoconstriction, increasing the blood pressure D. Increases the strength of myocardial contraction

A. Slows the heart rate

Which of the following statements regarding the AED and defibrillation is correct? A. The AED will not analyze the rhythm of a moving patient. B. Defibrillation is the first link in the American Heart Association chain of survival. C. The AED will shock any rhythm not accompanied by a pulse. D. CPR should be performed for 5 minutes before using the AED.

A. The AED will not analyze the rhythm of a moving patient.

A​ 65-year-old male complains of chest pain that started about 45 minutes ago and is radiating up to his jaw. He feels nauseous and has very little energy. His vitals are respirations of​ 24, pulse of​ 110, and BP of​ 88/60. What would make you decide to transport this​ patient? A. The low blood pressure reading B. The lack of energy C. The nausea D. The elevated pulse

A. The low blood pressure reading

In which of the following situations would the administration of aspirin to a cardiac patient be​ prohibited? A. The patient has a gastrointestinal ulcer. B. The patient feels dizzy. C. The patient does not currently take aspirin. D. The patient has a diastolic blood pressure greater than 90 mmHg.

A. The patient has a gastrointestinal ulcer.

One of the side effects of administering nitroglycerin to a cardiac patient could​ be: A. a drop in blood pressure. B. apnea. C. hypertension. D. chest pain.

A. a drop in blood pressure.

Which of the following is NOT a common sign or symptom associated with malfunction of an implanted cardiac pacemaker? A. a rapid heart rate B. syncope or dizziness C. heart rate less than 60 beats/min D. generalized weakness

A. a rapid heart rate

Which of the following MOST accurately describes a simple partial seizure? A. a seizure that begins in one extremity B. a seizure that causes the patient to stare blankly C. a seizure that is not preceded by an aura D. a generalized seizure without incontinence

A. a seizure that begins in one extremity

Which of the following MOST accurately describes a simple partial seizure? A. a seizure that begins in one extremity B. a seizure that causes the patient to stare blankly C. a seizure that is not preceded by an aura D. a generalized seizure without incontinence

A. a seizure that begins in one extremity

When caring for a patient with documented hypoglycemia, you should be MOST alert for: A. a seizure. B. an acute stroke. C. respiratory distress. D. a febrile convulsion.

A. a seizure.

A 66-year-old female with a history of hypertension and diabetes presents with substernal chest pressure of 2 hours' duration. Her blood pressure is 140/90 mm Hg, her pulse is 100 beats/min and irregular, and her respirations are 22 breaths/min with adequate depth. The patient does not have prescribed nitroglycerin, but her husband does. You should: A. administer oxygen, give her 324 mg aspirin, and assess her further. B. obtain a SAMPLE history and contact medical control for advice. C. give her 100% oxygen, attach the AED, and transport immediately. D. give her one nitroglycerin and reassess her systolic blood pressure.

A. administer oxygen, give her 324 mg aspirin, and assess her further.

Upon arriving at the residence of a patient with a possible cardiac problem, it is MOST important to: A. assess the scene for potential hazards. B. determine if you need additional help. C. request a paramedic unit for assistance. D. gain immediate access to the patient.

A. assess the scene for potential hazards.

You and your partner arrive at the scene of a middle-aged man who collapsed about 5 minutes ago. He is unresponsive, apneic, and pulseless. Bystanders are present, but have not provided any care. You should: A. begin high-quality CPR and apply the AED without delay. B. have your partner perform CPR while you question the bystanders. C. perform two-rescuer CPR for 5 minutes and request ALS backup. D. immediately apply the AED pads and analyze his cardiac rhythm.

A. begin high-quality CPR and apply the AED without delay.

Which of the following MOST accurately describes the cause of an ischemic stroke? A. blockage of a cerebral artery B. acute atherosclerotic disease C. rupture of a cerebral artery D. narrowing of a carotid artery

A. blockage of a cerebral artery

When an electrical impulse reaches the AV node, it is slowed for a brief period of time so that: A. blood can pass from the atria to the ventricles. B. blood returning from the body can fill the atria. C. the impulse can spread through the Purkinje fibers. D. the SA node can reset and generate another impulse.

A. blood can pass from the atria to the ventricles.

The most basic functions of the body, such as breathing, blood pressure, and swallowing, are controlled by the: A. brain stem. B. cerebrum. C. cerebellum. D. cerebral cortex.

A. brain stem.

The inferior vena cava returns deoxygenated blood to the right side of the heart from all of the following areas, EXCEPT the: A. brain. B. kidneys. C. abdomen. D. legs.

A. brain.

An​ 80-year-old male is presenting with pedal​ edema, JVD, and dyspnea. You should​ suspect: A. congestive heart failure. B. pulmonary embolism. C. dysrhythmia. D. multi-system trauma.

A. congestive heart failure.

Bradycardia

HR less than 60 BPM in adults. When the heart rate is slow, usually below 60 beats per minute.

Cardiogenic shock following AMI is caused by: A. decreased pumping force of the heart muscle. B. a profound increase in the patient's heart rate. C. hypovolemia secondary to severe vomiting. D. widespread dilation of the systemic vasculature.

A. decreased pumping force of the heart muscle.

Nitroglycerin relieves cardiac-related chest pain by: A. dilating the coronary arteries and improving cardiac blood flow. B. increasing the amount of work that is placed on the myocardium. C. contracting the smooth muscle of the coronary and cerebral arteries. D. constricting the coronary arteries and improving cardiac blood flow.

A. dilating the coronary arteries and improving cardiac blood flow.

In the cardiac conductive​ system, specialized muscles and​ __________ cause the heart to contract. A. electrical impulses B. the major blood vessels C. circulation between the heart and the lungs D. blood pressure

A. electrical impulses

A patient without a history of seizures experiences a sudden convulsion. The LEAST likely cause of this seizure is: A. epilepsy. B. a brain tumor. C. a serious infection. D. intracranial bleeding.

A. epilepsy.

The MOST common error associated with the use of the AED is: A. failure of the EMT to ensure the battery is charged. B. malfunction of the AED's internal computer processor. C. inappropriately placed adhesive defibrillation electrodes. D. inability of the EMT to recognize ventricular fibrillation.

A. failure of the EMT to ensure the battery is charged.

Risk factors for AMI that cannot be controlled include: A. family history. B. lack of exercise. C. excess stress. D. hyperglycemia.

A. family history.

The iliac arteries immediately subdivide into the: A. femoral arteries. B. peroneal arteries. C. anterior tibial arteries. D. posterior tibial arteries.

A. femoral arteries.

The spinal cord exits the cranium through the: A. foramen magnum. B. vertebral foramen. C. foramen lamina. D. cauda equina.

A. foramen magnum.

Signs and symptoms of a hypertensive emergency would MOST likely be delayed in patients who: A. have chronic hypertension. B. regularly take illegal drugs. C. have had a stroke in the past. D. are older than 40 years of age.

A. have chronic hypertension.

Major risk factors for AMI include all of the following, EXCEPT: A. hypoglycemia. B. hypertension. C. diabetes mellitus. D. elevated cholesterol.

A. hypoglycemia.

After the AED has delivered a shock, the EMT should: A. immediately resume CPR. B. assess for a carotid pulse. C. re-analyze the cardiac rhythm. D. transport the patient at once.

A. immediately resume CPR.

Which of the following would cause the greatest increase in cardiac output? A. increased heart rate and increased stroke volume B. decreased stroke volume and increased heart rate C. decreased heart rate and increased stroke volume D. decreased stroke volume and decreased heart rate

A. increased heart rate and increased stroke volume

Headache, vomiting, altered mental status, and seizures are all considered early signs of: A. increased intracranial pressure. B. decreased intracranial pressure. C. increased extracranial pressure. D. decreased extracranial pressure.

A. increased intracranial pressure.

An acute myocardial infarction (AMI) occurs when: A. myocardial tissue dies secondary to an absence of oxygen. B. the heart muscle progressively weakens and dysfunctions. C. coronary artery dilation decreases blood flow to the heart. D. the entire left ventricle is damaged and cannot pump blood.

A. myocardial tissue dies secondary to an absence of oxygen.

You are caring for a semiconscious man with left-sided paralysis. His airway is patent and his respirations are 14 breaths/min with adequate tidal volume. Treatment for this patient should include: A. oxygen via a nonrebreathing mask, left lateral recumbent position, and transport. B. assisted ventilation with a bag-mask device, right lateral recumbent position, and transport. C. an oral airway, assisted ventilation with a bag-mask device, Fowler's position, and transport. D. oxygen via a nonrebreathing mask, supine position with legs elevated 6″ to 12″, and transport.

A. oxygen via a nonrebreathing mask, left lateral recumbent position, and transport.

The EMT should use an AED on a child between 1 month and 8 years of age if: Select one: A. pediatric pads and an energy-reducing device are available. B. his or her condition is rapidly progressing to cardiac arrest. C. he or she is not breathing and has a weakly palpable pulse. D. special pads are used and the child has profound tachycardia.

A. pediatric pads and an energy-reducing

An absence seizure is also referred to as a: A. petit mal seizure. B. grand mal seizure. C. total body seizure. D. generalized motor seizure.

A. petit mal seizure.

Which of the following is a metabolic cause of a seizure? A. poisoning B. head trauma C. brain tumor D. massive stroke

A. poisoning

In the mnemonic​ OPQRST, the​ "P" stands​ for: A. provocation. B. pallor. C. perspiration. D. pressure.

A. provocation.

You are caring for a 68-year-old man with sudden onset of left-sided paralysis and slurred speech. His airway is patent, his respirations are 14 breaths/min with adequate depth, and his oxygen saturation is 98%. Treatment for this patient should include: A. recovery position and transport. B. ventilatory assistance and transport. C. high-flow oxygen and transport. D. oral glucose gel and transport.

A. recovery position and transport.

Which of the following is a blanket term used to represent any symptoms related to lack of oxygen in the heart muscle? A Heart attack B Myocardial infarction C Acute coronary syndrome D Angina pectoris

Acute coronary syndrome

Which of the following refers to any condition in which the heart may not be getting enough​ oxygen? A. Acute coronary syndrome B. Acute myocardial infarction C. Cardiac dysrhythmia D. Unstable angina

Acute coronary syndrome

You are assessing a 49-year-old man who complains of chest pressure that began the night before. He is conscious, but anxious, and tells you he has a history of angina and hypertension. After applying high-flow oxygen, you expose his chest to auscultate his lungs and note that he has a nitroglycerin patch on his right upper chest. His skin is cool and pale, his blood pressure is 78/50 mm Hg, and his pulse is 110 beats/min and irregular. You should: A. remove the nitroglycerin patch, place him in a supine position and elevate his lower extremities, and prepare for immediate transport. B. immediately remove the nitroglycerin patch, apply the AED in case he develops cardiac arrest, and transport to the closest hospital. C. move the nitroglycerin patch to the other side of his chest in case you need to apply the AED, keep him warm, and transport without delay. D. ask him if the nitroglycerin patch he is wearing has improved his chest pressure, complete your secondary assessment, and transport promptly.

A. remove the nitroglycerin patch, place him in a supine position and elevate his lower extremities, and prepare for immediate transport.

In contrast to monophasic defibrillation, biphasic defibrillation: A. requires a lower energy setting. B. delivers all shocks at 360 joules. C. begins with 300 joules and escalates. D. is only effective for ventricular tachycardia.

A. requires a lower energy setting.

Deoxygenated blood from the body returns to the: A. right atrium. B. left ventricle. C. right ventricle. D. left atrium.

A. right atrium.

Deoxygenated blood from the body returns to the: A. right atrium. B. right ventricle. C. left atrium. D. left ventricle.

A. right atrium.

A generalized seizure is characterized by: A. severe twitching of all the body's muscles. B. a blank stare and brief lapse of consciousness. C. unconsciousness for greater than 30 minutes. D. a core body temperature of greater than 103°F (40°C).

A. severe twitching of all the body's muscles.

The purpose of defibrillation is to: A. stop the chaotic, disorganized contraction of the cardiac cells. B. cause a rapid decrease in the heart rate of an unstable patient. C. improve the chance of cardiopulmonary resuscitation (CPR) being successful in resuscitation. D. prevent asystole from deteriorating into ventricular fibrillation.

A. stop the chaotic, disorganized contraction of the cardiac cells.

You are caring for a 70-year-old female with signs and symptoms of an acute stroke. She is conscious, has secretions in her mouth, and is breathing at a normal rate with adequate depth. You should: A. suction her oropharynx and apply 100% oxygen. B. insert an oral airway, apply oxygen, and transport. C. use a bag-mask device to assist her ventilations. D. place her on her side and prepare for rapid transport.

A. suction her oropharynx and apply 100% oxygen.

You are caring for a 70-year-old female with signs and symptoms of an acute stroke. She is conscious, has secretions in her mouth, is breathing at a normal rate with adequate depth, and has an oxygen saturation of 96%. You should: A. suction her oropharynx and apply 100% oxygen. B. insert an oral airway, apply oxygen, and transport. C. use a bag-mask device to assist her ventilations. D. place her on her side and prepare for rapid transport.

A. suction her oropharynx and transport immediately.

The key difference between angina pectoris and a myocardial infarction is​ that: A. the pain of angina pectoris usually goes away with rest. B. the pain of myocardial infarction usually goes away on its own. C. myocardial infarction is usually caused by stress. D. angina pectoris typically occurs at rest.

A. the pain of angina pectoris usually goes away with rest.

Prompt transport of a patient with a suspected AMI is important because: A. the patient may be eligible to receive thrombolytic therapy. B. 90% of the cardiac cells will die within the first 30 minutes. C. nitroglycerin can only be given in the emergency department. D. many patients with an AMI die within 6 hours.

A. the patient may be eligible to receive thrombolytic therapy.

When obtaining medical history information from the family of a suspected stroke patient, it is MOST important to determine: A. when the patient last appeared normal. B. if there is a family history of a stroke. C. if the patient has been hospitalized before. D. the patient's overall medication compliance.

A. when the patient last appeared normal.

Nitroglycerin is contraindicated in patients: A. who have experienced a head injury. B. with a history of an ischemic stroke. C. with a systolic blood pressure less than 120 mm Hg. D. who have taken up to two doses.

A. who have experienced a head injury.

With what condition does a patient often have a sense of impending doom?

ACS

Acute Myocardial Infarction

AKA: Heart attack - A condition in which a portion of the myocardium dies as a result of oxygen starvation

Lucid interval

After a head injury, a patient may have normal mental status for a period of time (usually minutes to hours), only to deteriorate markedly; period between injury and deterioration.

Which of the following BEST describes the continued need for prehospital advanced cardiac life support, even when EMTs in the community carry AEDs? Not al l cardiac arrests are due to problems that respond to defibrillation. The patient may need medications to support his cardiac rhythm and blood pressure following successful defibrillation. In cases of refractory or recurrent ventricular fibrillation, the use of medications may increase the chances of defibrillating successfully. All of the above

All of the above

Which of the following may be a symptom of a problem with the heart? Mild chest discomfort Severe, crushing pain in the chest Nausea, with or without vomiting All of the above

All of the above

Which of the following is NOT typical of angina pectoris? Responds to nitroglycerin Lasts 3 to 5 minutes Often subsides with rest All of the above are typical findings of angina pectoris.

All of the above are typical findings of angina pectoris.

High-flow oxygen.

All patients with altered mental status, regardless of cause, should receive...

What is an AMI?

An Acute Myocardial Infarction, a condition is which a portion of the myocardium dies as a result of oxygen starvation

arrhythmia

An abnormal heart rhythm

Hypoglycemia

An abnormally low blood glucose level.

Bacterial meningitis

An inflammation of the meninges caused by bacterial infection, is a central nervous system infection in which the patient may complain of a headache, stiff neck, fever, and sensitivity to light.

Cerebrovascular Accident (CVA)

An interruption of blood flow 2 the brain that results in the loss of brain function. Also called a stroke.

STROKE

An interruption of blood flow 2 the brain that results in the loss of brain function;also called a cerebrovascular accident (CVA).

A weakened area of an artery that balloons out and may rupture, causing catastrophic bleeding, is called:

Aneurysm.

Which of the following is a major difference between angina pectoris and AMI?

Anginal pain typically subsides with rest.

arteriosclerosis

Hardening of the arteries

Altered mental status

Any deviation from alert and oriented to person, place, time, and event, or any deviation from a patient's normal baseline mental status.

You are on the scene of a 78-year-old male patient complaining of posterior chest pain that he states feels "like someone is tearing out my back." What condition do you suspect?

Aortic aneurysm

Which of the following skills is typically NOT used to manage the airway and breathing of a patient in cardiac​ arrest? A. Opening the airway manually B. Suctioning C. Insertion of an oropharyngeal airway D. Application of oxygen via nasal cannula

Application of oxygen via nasal cannula

Your patient is a​ 44-year-old female who has collapsed while jogging. She has been unresponsive for 4 to 5 minutes by the time you arrive. Her husband appears to be performing​ high-quality CPR. What should be your first​ action?

Apply the AED

Your patient is a 44-year-old female who has collapsed while jogging. She has been unresponsive for 4 to 5 minutes by the time you arrive. Her husband appears to be performing high-quality CPR. Which of the following should be your first action?

Apply the AED.

You are the first on the scene of a 72-year-old patient in cardiac arrest. You have your medical supply kit, oxygen, and an AED. At least 4 to 5 minutes of high-quality CPR has been provided by the police officer who arrived before you. You have confirmed an open airway, apnea, and pulselessness. Which of the following should you do next?

Apply the defibrillator pads and shock as indicated.

Your patient is a 60-year-old male who is complaining of severe chest pain and difficulty breathing. He is pale, sweaty, and pleads with you, "Don't let me die, I think I'm going to die." Which of the following measures is NOT part of your initial treatment of this patient?

Applying the defibrillator pads to his chest

Your patient is a 59-year-old man who has collapsed while working in the yard. He has been unresponsive for 4 to 5 minutes by the time you arrive. His neighbor is attempting CPR, but the quality is questionable. Which of the following should be your first action?

Ask the neighbor to stop CPR so you can check the patient's pulse.

Your patient is a 66-year-old female who has regained a pulse after a shock with an AED; however, she remains unresponsive and is not breathing adequately. Which of the following should be done next?

Assist ventilations with a bag-valve-mask device and supplemental oxygen and anticipate that the patient may go back into cardiac arrest.

Which of the following refers to chest pain that occurs when blood supply to the heart is reduced and a portion of the heart muscle is not receiving enough​ oxygen? A. Aneurysm B. Angina C. Acute myocardial infarction D. Hypertension

B. Angina

A 30-year-old male experienced a generalized (tonic-clonic) seizure, which stopped before you arrived at the scene. The patient is conscious, is answering your questions appropriately, and refuses EMS transport. Which of the following would be the MOST compelling reason to disagree with his refusal of transport? A. He has experienced seizures since he was 20 B. He is currently not prescribed any medications C. His wife states that this was his "usual" seizure D. His Glasgow Coma Scale (GCS) score is 15

B. He is currently not prescribed any medications

Your patient is a​ 66-year-old female who has regained a pulse after a shock with an​ AED; however, she remains unresponsive and is not breathing adequately. Which of the following should be done​ next? A. Remove the AED and assist the​ patient's ventilations with a​ bag-valve mask and supplemental​ oxygen, and keep reassessing the pulse. B. Assist ventilations with a​ bag-valve-mask device and supplemental oxygen and anticipate that the patient may go back into cardiac arrest. C. Deliver a fourth shock to improve the​ patient's respiratory status. D. Apply a nonrebreather mask with​ high-concentration oxygen and keep reassessing the pulse.

B. Assist ventilations with a​ bag-valve-mask device and supplemental oxygen and anticipate that the patient may go back into cardiac arrest.

Which of the following types of defibrillators will adjust the amount of energy used on the basis of​ impedance? A. Monophasic AED B. Biphasic AED C. Implanted defibrillator D. Triphasic defibrillator

B. Biphasic AED

Delivery of an electrical shock to restore a normal heart rhythm most directly affects which of the​ following? A. Peripheral pulses B. Cardiac conduction system C. Blood pressure D. Blood circulation

B. Cardiac conduction system

What is not a result of a mechanical malfunction of the​ heart? A. Shock B. Dysrhythmia C. Pulmonary edema D. Cardiac arrest

B. Dysrhythmia

Which of the following statements regarding angina pectoris is​ TRUE? A. It generally lasts 30 to 60 minutes. B. It can be brought on by exertion or stress. C. It results in death of a smaller portion of myocardium than does a heart attack. D. It is generally relieved by​ over-the-counter medications.

B. It can be brought on by exertion or stress.

Which of the following is the result of a portion of the heart muscle dying due to a lack of​ oxygen? A. Heart failure B. Myocardial infarction C. Angina pectoris D. Cardiac arrest

B. Myocardial infarction

After placing a patient experiencing chest pain in a comfortable​ position, it is most important for you to assess the need for administering which of the​ following? A. Epinephrine​ auto-injector ​(Epi-Pené​) B. Oxygen C. Albuterol D. Aspirin

B. Oxygen

You are on scene with a​ 48-year-old unresponsive male patient. Bystanders state he complained of chest pain and then suddenly collapsed. Vital signs are blood pressure​ 68/42, pulse​ 36, and respiratory rate 3. He is unresponsive to painful stimuli. After performing the primary assessment and treating all life​ threats, what is your next​ intervention? A. Insert an oral airway. B. Package the patient for rapid transport. C. Apply the AED. D. Administer​ high-concentration oxygen at 15 lpm by​ bag-valve mask.

B. Package the patient for rapid transport.

Which of the following is the beneficial action of a beta blocker​ medication? A. Causes​ vasoconstriction, increasing the blood pressure B. Slows the heart rate C. Increases the strength of myocardial contraction D. Increases the amount of oxygen needed by the myocardium

B. Slows the heart rate

Which of the following cardiac dysrhythmias cannot produce a​ pulse? A. Tachycardia B. Ventricular fibrillation C. Ventricular dysrhythmia D. Bradycardia

B. Ventricular fibrillation

You must reassess your patient within two minutes after administering​ nitroglycerin, as one of the side effects​ is: A. numbness and tingling in the extremities. B. a decrease in blood pressure. C. a brief feeling of shortness of breath. D. a sudden decrease in heart rate.

B. a decrease in blood pressure.

Administer oxygen to your chest pain patient to maintain an oxygen saturation level​ of: A. up to 100 percent. B. at least 94 percent. C. at least 90 percent. D. up to 94 percent.

B. at least 94 percent.

Assuming your protocol allows the administration of nitroglycerin when certain conditions​ exist, what is the maximum number of tablets to be administered in the prehospital​ setting? A. 1 B. 3 C. 4 D. 2

B. 3

What Glasgow Coma Scale (GCS) score would you assign to a patient who responds to painful stimuli, uses inappropriate words, and maintains his or her arms in a flexed position? A. 6 B. 8 C. 9 D. 10

B. 8

Which of the following refers to chest pain that occurs when blood supply to the heart is reduced and a portion of the heart muscle is not receiving enough​ oxygen? A. Aneurysm B. Angina pectoris C. Hypertension D. Acute myocardial infarction

B. Angina pectoris

Which of the following is a major difference between angina pectoris and AMI? A. AMI is caused by myocardial ischemia. B. Anginal pain typically subsides with rest. C. Nitroglycerin has no effect on angina pectoris. D. Pain from an AMI subsides within 30 minutes.

B. Anginal pain typically subsides with rest.

Which of the following is a general term used to refer to a problem with the​ heart? A. Cardiac dysrhythmia B. Cardiac compromise C. Congestive heart failure D. Myocardial infarction

B. Cardiac compromise

You respond to the scene of a​ 56-year-old obese female complaining of respiratory distress. She states that she has been feeling weak and a​ "little sick" for the past two​ days, but the respiratory distress has been getting progressively worse for the past several hours. She states she has​ "heart problems," suffers from high blood​ pressure, and takes a​ "water pill." She is afebrile and has coarse crackles bilaterally. What is most likely the cause of her respiratory​ distress? A. Aortic aneurysm B. Congestive heart failure C. Dysrhythmia D. Acute myocardial infarction

B. Congestive heart failure

What is not a result of a mechanical malfunction of the​ heart? A. Pulmonary edema B. Dysrhythmia C. Cardiogenic shock D. Cardiac arrest

B. Dysrhythmia

Muscle control and body coordination are controlled by the: A. cerebrum. B. cerebellum. C. brain stem. D. cerebral cortex.

B. cerebellum.

In patients with pulmonary hypertension, the right ventricular stroke work index will be:

Increased

You are treating a male patient with chest pain caused by the complete occlusion of one of the coronary arteries. What would you most likely expect with this​ patient? A. His pain will go away with oxygen. B. His pain will not be alleviated with any of the above medications. C. His pain will be reduced with aspirin. D. His pain will be alleviated with nitroglycerin.

B. His pain will not be alleviated with any of the above medications.

Which of the following conditions would be the LEAST likely to mimic the signs and symptoms of a stroke? A. hypoglycemia B. hypovolemia C. a postictal state D. intracranial bleeding

B. Hypovolemia

What is the function of the left atrium? A. It ejects oxygenated blood into the aorta. B. It receives oxygenated blood from the lungs. C. It receives blood from the pulmonary arteries. D. It receives oxygenated blood from the vena cava.

B. It receives oxygenated blood from the lungs.

You are called to a home and find a 56-year-old woman supine in her bed. She appears alert, but has slurred speech. Her family tells you she has a history of TIAs and hypertension. You ask the patient, "What day is it today?" Her reply is, "Butterfly." Which area of the brain is likely affected? A. Occipital lobe B. Left hemisphere C. Cerebellum D. Right hemisphere

B. Left hemisphere

Which of the following best describes a fluttering sensation in the​ chest? A. Tachycardia B. Palpitations C. Pulseless electrical activity of the heart D. Dysrhythmia

B. Palpitations

Which of the following is the name given to the condition in which fluid accumulates in the​ lungs? A. Pedal edema B. Pulmonary edema C. Congestive heart failure D. Dyspnea

B. Pulmonary edema

Which of the following is not a typical indication of congestive heart failure​ (CHF)? A. Increased heart rate B. Weight loss C. Jugular venous distension D. Productive cough

B. Weight loss

Which of the following patients would MOST likely demonstrate typical signs of infection, such as a fever? A. a 3-month-old female who was born prematurely B. a 17-year-old male with depression and anxiety C. a 35-year-old female in the later stages of AIDS D. an 88-year-old male with chronic renal problems

B. a 17-year-old male with depression and anxiety

Acute coronary syndrome (ACS) is a term used to describe: A. the warning signs that occur shortly before a heart attack. B. a group of symptoms that are caused by myocardial ischemia. C. a severe decrease in perfusion caused by changes in heart rate. D. the exact moment that a coronary artery is completely occluded.

B. a group of symptoms that are caused by myocardial ischemia.

You are assessing a 49-year-old man who, according to his wife, experienced a sudden, severe headache and then passed out. He is unresponsive and has slow, irregular breathing. His blood pressure is 190/94 mm Hg and his pulse rate is 50 beats/min. His wife tells you that he has hypertension and diabetes. He has MOST likely experienced: A. acute hypoglycemia. B. a ruptured cerebral artery. C. a complex partial seizure. D. an occluded cerebral artery.

B. a ruptured cerebral artery.

An area of swelling or enlargement in a weakened arterial wall is called: A. a thrombus. B. an aneurysm. C. an embolism. D. atherosclerosis.

B. an aneurysm.

Your patient has a weakened section of the arterial wall that is ballooning. This is known as​ a(n): A. embolism. B. aneurysm. C. thrombus. D. occlusion.

B. aneurysm.

The MOST common reason that many people experiencing AMI do not seek immediate medical attention is because they: A. are elderly. B. are in denial. C. cannot afford it. D. do not trust EMTs.

B. are in denial.

Which of the following medications is commonly given to patients with chest pain to prevent blood clots from forming or getting bigger? A. furosemide (Lasix) B. aspirin C. oxygen D. digoxin (Lanoxin)

B. aspirin

Administer oxygen to your chest pain patient to maintain an oxygen saturation level​ of: A. up to​ 94%. B. at least​ 94%. C. up to​ 100%. D. at least​ 90%.

B. at least​ 94%.

You and your partner arrive at the scene of a middle-aged man who collapsed about 5 minutes ago. He is unresponsive, apneic, and pulseless. Bystanders are present, but have not provided any care. You should: A. immediately apply the AED pads and analyze his cardiac rhythm. B. begin high-quality CPR and apply the AED as soon as possible. C. have your partner perform CPR while you question the bystanders. D. perform two-rescuer CPR for 5 minutes and request ALS backup.

B. begin high-quality CPR and apply the AED as soon as possible.

You are dispatched to a residence for a 56-year-old male with an altered mental status. Upon arrival at the scene, the patient's wife tells you that he complained of chest pain the day before, but would not allow her to call EMS. The patient is semiconscious; has rapid, shallow respirations; and has a thready pulse. You should: A. obtain baseline vital signs. B. begin ventilatory assistance. C. attach the automated external defibrillator (AED) immediately. D. apply a nonrebreathing mask.

B. begin ventilatory assistance.

Rapid, labored breathing in a patient with signs and symptoms of AMI should make you suspicious for: A. a cardiac arrhythmia. B. congestive heart failure. C. significant hypotension. D. right ventricular failure.

B. congestive heart failure.

Which of the following is NOT a function of the sympathetic nervous system? A. dilation of blood vessels in the muscles B. constriction of blood vessels in the muscles C. increases in the heart and respiratory rates D. constriction of blood vessels in the digestive system

B. constriction of blood vessels in the muscles

Conditions that narrow or block the arteries of the heart are​ called: A. cardiac dysrhythmia. B. coronary artery disease. C. pulmonary edema. D. pulmonary embolism.

B. coronary artery disease.

Ischemic heart disease is MOST accurately defined as: A. absent myocardial blood flow due to a blocked coronary artery. B. decreased blood flow to one or more portions of the myocardium. C. death of a portion of the heart muscle due to a decrease in oxygen. D. decreased blood flow to the heart muscle due to coronary dilation.

B. decreased blood flow to one or more portions of the myocardium.

Which of the following signs is commonly observed in patients with right-sided heart failure? A. labored breathing B. dependent edema C. pulmonary edema D. flat jugular veins

B. dependent edema

A 66-year-old woman presents with a stabbing pain in the middle of her chest that radiates to her back. She tells you that the pain suddenly began about 30 minutes ago and has been severe since the onset. She has a history of hypertension, but admits to being noncompliant with her antihypertensive medications. When you assess her, you find that her blood pressure is significantly higher in her left arm than it is in her right arm. What are her signs and symptoms MOST indicative of? A. unstable angina B. dissecting aortic aneurysm C. AMI D. hypertensive emergency

B. dissecting aortic aneurysm

Prior to attaching the AED to a cardiac arrest patient, the EMT should: Select one: A. perform CPR for 30 seconds. B. dry the chest if it is wet. C. assess for a pulse for 20 seconds. D. contact medical control.

B. dry the chest if it is wet.

Prior to attaching the AED to a cardiac arrest patient, the EMT should: A. contact medical control. B. dry the chest off if it is wet. C. perform CPR for 30 seconds. D. assess for a pulse for 20 seconds.

B. dry the chest off if it is wet.

A patient whose speech is slurred and difficult to understand is experiencing: A. aphasia. B. dysarthria. C. dysphagia. D. paraplegia.

B. dysarthria.

A patient whose speech is slurred and difficult to understand is experiencing: A. aphasia. B. dysarthria. C. dysphagia. D. paraplegia.

B. dysarthria.

The anterior aspect of the cerebrum controls: A. touch. B. emotion. C. movement. D. vision.

B. emotion.

Blood that is ejected from the right ventricle: A. enters the systemic circulation. B. flows into the pulmonary arteries. C. has a high concentration of oxygen. D. was received directly from the aorta.

B. flows into the pulmonary arteries.

Patients with a subarachnoid hemorrhage typically complain of a sudden severe: A. bout of dizziness. B. headache. C. altered mental status. D. thirst.

B. headache.

Which of the following medications is NOT used to treat patients with a history of seizures? A. phenytoin (Dilantin) B. hydromorphone (Dilaudid) C. carbamazepine (Tegretol) D. phenobarbital

B. hydromorphone (Dilaudid)

Common side effects of nitroglycerin include all of the following, EXCEPT: A. bradycardia. B. hypertension. C. hypotension. D. severe headache.

B. hypertension.

The MOST significant risk factor for a hemorrhagic stroke is: A. severe stress. B. hypertension. C. heavy exertion. D. diabetes mellitus

B. hypertension.

The MOST significant risk factor for a hemorrhagic stroke is: A. severe stress. B. hypertension. C. heavy exertion. D. diabetes mellitus.

B. hypertension.

Which of the following signs or symptoms would you NOT expect to encounter in a patient with congestive heart failure? A. hypertension and tachycardia B. hypotension and flat jugular veins C. the presence of rales in the lungs D. trouble breathing while lying down

B. hypotension and flat jugular veins

Which of the following conditions would be the LEAST likely to mimic the signs and symptoms of a stroke? A. hypoglycemia B. hypovolemia C. a postictal state D. intracranial bleeding

B. hypovolemia

After the AED has delivered a shock, the EMT should: A. assess for a carotid pulse. B. immediately resume CPR. C. reanalyze the cardiac rhythm. D. transport the patient at once.

B. immediately resume CPR.

You are dispatched to a residence for a 66-year-old male who, according to family members, has suffered a massive stroke. Your primary assessment reveals that the patient is unresponsive, apneic, and pulseless. You should: A. assess the patient for a facial droop and hemiparesis. B. initiate cardiopulmonary resuscitation (CPR) and attach an automated external defibrillator (AED) as soon as possible. C. obtain a blood glucose sample to rule out hypoglycemia. D. perform CPR for 5 minutes before applying the AED.

B. initiate cardiopulmonary resuscitation (CPR) and attach an automated external defibrillator (AED) as soon as possible.

An insufficient supply of oxygenated blood to an area of the body is​ called: A. dyspnea. B. ischemia. C. embolism. D. coronary artery disease.

B. ischemia.

The AED is MOST advantageous to the EMT because: A. it is lightweight, easy to use, and safe for the EMT who is using it. B. it delivers prompt defibrillation to patients with ventricular fibrillation. C. its use does not require the presence of ALS personnel. D. it delivers an unlimited number of shocks with the same amount of energy

B. it delivers prompt defibrillation to patients with ventricular fibrillation.

The pressure backup from the vena cava in​ right-sided congestive heart failure causes the sign​ of: A. shortness of breath. B. jugular vein distension. C. tachypnea. D. elevated blood pressure.

B. jugular vein distension.

A patient with atherosclerotic heart disease experiences chest pain during exertion because the: A. coronary arteries suddenly spasm and cause a marked reduction in myocardial blood flow. B. lumen of the coronary artery is narrowed and cannot accommodate increased blood flow. C. tissues of the myocardium undergo necrosis secondary to a prolonged absence of oxygen. D. ragged edge of a tear in the coronary artery lumen causes local blood clotting and arterial narrowing.

B. lumen of the coronary artery is narrowed and cannot accommodate increased blood flow.

The principal difference between a patient who has had a stroke and a patient with hypoglycemia almost always has to do with the A. papillary response. B. mental status. C. blood pressure. D. capillary refill time.

B. mental status.

Angina pectoris occurs when: A. a coronary artery is totally occluded by plaque. B. myocardial oxygen demand exceeds the supply. C. one or more coronary arteries suddenly spasm. D. myocardial oxygen supply exceeds the demand.

B. myocardial oxygen demand exceeds the supply.

You arrive at a grocery store shortly after a 35-year-old male stopped seizing. Your assessment reveals that he is confused and incontinent of urine. The patient's girlfriend tells you that he has a history of seizures and takes carbamazepine (Tegretol). When obtaining further medical history from the girlfriend, it is MOST important to: A. determine if the patient is a known alcohol abuser. B. obtain a description of how the seizure developed. C. determine when he was last seen by his physician. D. ask her how long the patient has been taking his medication.

B. obtain a description of how the seizure developed.

Prior to assisting a patient with his or her prescribed nitroglycerin, the EMT must: A. ensure the medication is in tablet form. B. obtain authorization from medical control. C. determine who prescribed the nitroglycerin. D. wait at least 5 minutes after assessing the blood pressure.

B. obtain authorization from medical control.

In contrast to AMI, a dissecting aortic aneurysm: A. is more commonly associated with pressure in the chest. B. often presents with pain that is maximal from the onset. C. usually presents gradually, often over a period of hours. D. is typically preceded by other symptoms, such as nausea.

B. often presents with pain that is maximal from the onset.

The heart has no reserve of​ __________, so any interruption in its supply results in significant problems. A. blood B. oxygen C. nutrients D. carbon dioxide

B. oxygen

Your EMS team is performing CPR on a 60-year-old male in cardiac arrest. You connect the AED, push the analyze button, and receive a "no shock advised" message. You should: A. reanalyze the patient's cardiac rhythm. B. perform CPR for 2 minutes and reassess. C. determine if a palpable pulse is present. D. immediately assess the patient's airway.

B. perform CPR for 2 minutes and reassess.

After assisting your patient with his or her nitroglycerin, you should: A. place the patient in a recumbent position in case he or she faints. B. reassess the blood pressure within 5 minutes to detect hypotension. C. avoid further dosing if the patient complains of a severe headache. D. perform a secondary assessment before administering further doses.

B. reassess the blood pressure within 5 minutes to detect hypotension.

When transporting a stable stroke patient with a paralyzed extremity, place the patient in a: A. recumbent position with the paralyzed side up. B. recumbent position with the paralyzed side down. C. sitting position with the head at a 45° to 90° angle. D. supine position with the legs elevated 6″ to 12″.

B. recumbent position with the paralyzed side down.

When transporting a stable stroke patient with unilateral paralysis, it is best to place the patient in a: A. recumbent position with the paralyzed side up. B. recumbent position with the paralyzed side down. C. sitting position with the head at a 45° to 90° angle. D. supine position with the legs elevated 6″ to 12″.

B. recumbent position with the paralyzed side down.

You are assessing the arm drift component of the Cincinnati Prehospital Stroke Scale on a 60-year-old woman. When she holds both of her arms out in front of her and closes her eyes, both of her arms immediately fall to her sides. You should: A. repeat the arm drift test and ensure that her palms are facing downward. B. repeat the arm drift test, but move the patient's arms into position yourself. C. instruct the patient to keep her eyes open and then repeat the arm drift test. D. defer this part of the test and assess her for facial droop and slurred speech.

B. repeat the arm drift test, but move the patient's arms into position yourself.

You are assessing the arm drift component of the Cincinnati Prehospital Stroke Scale on a 60-year-old woman. When she holds both of her arms out in front of her and closes her eyes, both of her arms immediately fall to her sides. You should: A. repeat the arm drift test and ensure that her palms are facing downward. B. repeat the arm drift test, but move the patient's arms into position yourself. C. instruct the patient to keep her eyes open and then repeat the arm drift test. D. defer this part of the test and assess her for facial droop and slurred speech.

B. repeat the arm drift test, but move the patient's arms into position yourself.

The right coronary artery supplies blood to the: A. left ventricle and inferior wall of the right atrium. B. right ventricle and inferior wall of the left ventricle. C. right atrium and posterior wall of the right ventricle. D. left ventricle and posterior wall of the right ventricle.

B. right ventricle and inferior wall of the left ventricle.

Which of the following conditions is NOT a common cause of seizures? A. poisoning or overdose B. severe hypovolemia C. acute hypoglycemia D. acute alcohol withdrawal

B. severe hypovolemia

A 67-year-old female with severe chest pain becomes unresponsive, pulseless, and apneic during transport. You should: A. defibrillate with the AED while continuing transport to the hospital. B. stop the ambulance, begin CPR, and attach the AED as soon as possible. C. perform CPR for 1 to 2 minutes and then analyze her rhythm with an AED. D. alert the receiving hospital and perform CPR for the duration of the transport.

B. stop the ambulance, begin CPR, and attach the AED as soon as possible.

When the myocardium requires more oxygen: A. the heart contracts with less force. B. the arteries supplying the heart dilate. C. the heart rate decreases significantly. D. the AV node conducts fewer impulses.

B. the arteries supplying the heart dilate.

A dissecting aortic aneurysm occurs when: A. a weakened area develops in the aortic wall. B. the inner layers of the aorta become separated. C. the aorta ruptures, resulting in profound bleeding. D. all layers of the aorta suddenly contract.

B. the inner layers of the aorta become separated.

A patient with atherosclerotic heart disease experiences chest pain during exertion because: A. the ragged edge of a tear in the coronary artery lumen causes local blood clotting and arterial narrowing. B. the lumen of the coronary artery is narrowed and cannot accommodate increased blood flow. C. tissues of the myocardium undergo necrosis secondary to a prolonged absence of oxygen. D. the coronary arteries suddenly spasm and cause a marked reduction in myocardial blood flow.

B. the lumen of the coronary artery is narrowed and cannot accommodate increased blood flow.

Most patients are instructed by their physician to take up to _______ doses of nitroglycerin before calling EMS. A. two B. three C. four D. five

B. three

You respond to a residence for a child who is having a seizure. Upon arrival at the scene, you enter the residence and find the mother holding her child, a 2-year-old male. The child is conscious and crying. According to the mother, the child had been running a high fever and then experienced a seizure that lasted approximately 3 minutes. You should: A. cover the child with wet towels and give oxygen via nasal cannula. B. transport the child to the hospital and reassure the mother en route. C. advise the mother to take her child to the doctor the following day. D. call medical control and request permission to give the child aspirin.

B. transport the child to the hospital and reassure the mother en route.

You respond to a residence for a child who is having a seizure. Upon arrival at the scene, you enter the residence and find the mother holding her child, a 2-year-old male. The child is conscious and crying. According to the mother, the child had been running a high fever and then experienced a seizure that lasted approximately 3 minutes. You should: A. cover the child with wet towels and give oxygen via nasal cannula. B. transport the child to the hospital and reassure the mother en route. C. advise the mother to take her child to the doctor the following day. D. call medical control and request permission to give the child aspirin.

B. transport the child to the hospital and reassure the mother en route.

The descending aorta divides into the two iliac arteries at the level of the: A. nipple line. B. umbilicus. C. iliac crest. D. pubic symphysis.

B. umbilicus.

When documenting a patient's description of his or her chest pain or discomfort, the EMT should: A. use medical terminology. B. use the patient's own words. C. underline the patient's quotes. D. document his or her own perception.

B. use the patient's own words.

Nitroglycerin is contraindicated in patients: A. who have taken up to two doses. B. who have experienced a head injury. C. with a history of an ischemic stroke. D. with a systolic blood pressure less than 120 mm Hg.

B. who have experienced a head injury.

What are the contraindications of nitroglycerin?

BP is less than 90 Patient has head injury Patient is infant or child Patient has already taken max allowable dose Patient had recently taken viagra, cialis, levitra or other ED medication

Epidural bleeding

Bleeding outside the dura and under the skull.

occlusion

Blockage in a blood vessel, is AKA.

Sudden death following AMI is MOST often caused by: A. cardiogenic shock. B. severe bradycardia. C. ventricular fibrillation. D. congestive heart failure.

C. ventricular fibrillation.

What is an embolism?

Blockage of a vessel by a clot or foreign material brought to the site the blood current

Occlusion

Blockage, as of an artery by fatty deposits

The periodic, noninvasive measurement of cardiac performance that requires only the use of sphygmomanometer is called?

Blood Pressure >>A new method has been developed that estimates CO using pulse - waveform (PWA) from a brachial cuff sphygmomanometer.

Which of the following is the way in which the heart muscle receives oxygen?

Blood from the aorta enters the coronary arteries.

Which of the following would be the result of an obstruction in a branch of the pulmonary artery?

Blood is not pumped to the lungs.

Automatic defibrillation is not appropriate in most cases of infant cardiac arrest due to which of the following? Ventricular fibrillation is not the primary cause of cardiac arrest in the pediatric patient. primary cause of cardiac arrest in the pediatric patient. The energy delivered by the AED is not appropriate to a smaller patient. Both A and B are correct. Neither A nor B is correct.

Both A and B are correct.

Blood flow is directed to organs that have the highest metabolic needs when at rest. Which organ would have the lowest metabolic needs when at rest?

Brain, heart, skin, and others (muscles, liver, and kidneys receive the greatest amount)

atherosclerosis

Buildup of plaque in the blood vessels

RCA

The heart artery that feeds the right ventricle, atrium and septum.

What is the dosage for aspirin given to treat acute coronary​ syndrome? A. 40 mg of coated adult aspirin B. 81 mg of chewable baby aspirin C. 162 to 324 mg of chewable baby aspirin D. 81 mg of coated adult aspirin

C. 162 to 324 mg of chewable baby aspirin

Which of the following patients should have an automated external defibrillator​ applied? A. A​ 40-year-old female in cardiac arrest due to chest injuries sustained in a motor vehicle crash B. A​ 67-year-old man with​ severe, crushing chest pain that is not relieved by nitroglycerin C. A​ 19-year-old college athlete who collapsed during football practice and is pulseless and apneic D. A​ 6-month-old in severe respiratory distress

C. A​ 19-year-old college athlete who collapsed during football practice and is pulseless and apneic

Which of the following is the general term used to refer to a problem with the​ heart? A. Congestive heart failure B. Cardiac dysrhythmia C. Cardiac compromise D. Myocardial infarction

C. Cardiac compromise

Which of the following BEST describes the​ EMT's intended role in the ideal provision of emergency cardiac​ care? A. Early CPR B. Early notification C. Early defibrillation D. Early advanced cardiac life support

C. Early defibrillation

Which of the following would NOT be considered an element of​ post-cardiac arrest​ care? A. Maintaining adequate oxygenation B. ​12-lead ECG C. Early defibrillation D. Inducing hypothermia

C. Early defibrillation

Which of the following is the name given to the condition in which fluid accumulates in the​ lungs? A. Chronic obstructive pulmonary disease​ (COPD) B. Dyspnea C. Pulmonary edema D. Pedal edema

C. Pulmonary edema

You are transporting a​ 50-year-old male patient whom you successfully defibrillated at the scene. You are five minutes away from the hospital when the patient goes back into cardiac arrest. Which of the following is the best course of​ action? A. Initiate CPR and continue transporting. B. Tell the driver to stop and assist you with​ CPR, and request another unit for assistance. C. Tell the driver to​ stop, analyze the cardiac​ rhythm, and deliver a shock as necessary. D. Analyze the cardiac rhythm and deliver shocks as necessary.

C. Tell the driver to​ stop, analyze the cardiac​ rhythm, and deliver a shock as necessary.

A​ 65-year-old male complains of chest pain that started about 45 minutes ago and is radiating up to his jaw. He feels nauseous and has very little energy. His vitals are respirations of​ 24, pulse of​ 110, and BP of​ 88/60. What would make you decide to transport this​ patient? A. The elevated pulse B. The nausea C. The low blood pressure reading D. The lack of energy

C. The low blood pressure reading

Which of the following BEST describes the pain associated with acute coronary​ syndrome? A. The pain will go away after a few hours. B. The pain is a pressure in the chest. C. The pain may be chest​ pressure, mild​ discomfort, or not present at all. D. The pain is only a mild discomfort.

C. The pain may be chest​ pressure, mild​ discomfort, or not present at all.

To obtain the coronary perfusion pressure, what two values are needed?

CPP = Diastolic BP - PCWP NI: 60 to 80 mm Hg

Which of the following is NOT a reason that an AED may indicate that there is​ "no shock​ advised"? A. The patient has organized electrical activity in the heart but no pulse. B. The​ patient's heart has no electrical​ activity; he is​ "flat line," or in asystole. C. The patient is in ventricular fibrillation or pulseless ventricular tachycardia. D. The​ patient's heart rhythm is normal.

C. The patient is in ventricular fibrillation or pulseless ventricular tachycardia.

What is the MOST common reason for the​ AED's delivering a shock​ inappropriately? A. Electrical interference B. Mechanical error C. User error D. Software error

C. User error

As you arrive for a patient in cardiac​ arrest, bystanders are analyzing the patient with an AED. You​ should: A. stay back until bystanders transfer care to you. B. turn off the AED and check for a pulse. C. allow the AED to shock before you take over care. D. remove the AED and apply your own.

C. allow the AED to shock before you take over care.

For both the Lucas and​ Auto-Pulse CPR​ devices, you would take Standard Precautions​ and: A. apply the stabilization strap. B. position the pressure pad so that it touches the chest. C. ensure CPR is in progress and effective. D. close the Lifeband over the​ patient's chest.

C. ensure CPR is in progress and effective.

You have just used an AED to deliver a shock to your cardiac arrest patient. You should​ immediately: A. begin artificial ventilations. B. ​re-analyze with the AED. C. resume chest compressions. D. check a carotid pulse.

C. resume chest compressions.

The Lucas CPR device​ involves: A. providing​ bag-mask ventilation at a rate of two ventilations for every 30 compressions. B. removing the stabilization strap if the patient must be moved. C. stopping CPR just long enough to put a base plate under the patient. D. closing the Lifeband chest band over the​ patient's chest.

C. stopping CPR just long enough to put a base plate under the patient.

What is the dosage for aspirin given to treat acute coronary​ syndrome? A. 81 mg of chewable baby aspirin B. 40 mg of coated adult aspirin C. 162-324 mg of chewable baby aspirin D. 81 mg of coated adult aspirin

C. 162-324 mg of chewable baby aspirin

Your patient opens his eyes, moans, and pulls away from you when you pinch his trapezius muscle. You should assign a Glasgow Coma Scale (GCS) score of: A. 6. B. 7. C. 8. D. 9.

C. 8.

Which of the following refers to any condition in which the heart may not be getting enough​ oxygen? A. Angina pectoris B. Acute myocardial infarction C. Acute coronary syndrome D. Cardiac dysrhythmia

C. Acute coronary syndrome

Which of the following terms is used to represent any symptoms that are related to a lack of oxygen in the heart​ muscle? A. Congestive heart failure B. Angina pectoris C. Acute coronary syndrome D. Coronary artery disease

C. Acute coronary syndrome

Which of the following represents the MOST appropriate method of assisting a patient with his or her prescribed nitroglycerin tablet or spray? A. Encourage the patient to chew the tablet to increase its effectiveness. B. Place the medication under the tongue and have the patient swallow it. C. Administer the medication sublingually and allow it to dissolve or absorb. D. Wait 15 minutes and reassess the blood pressure prior to administering another dose.

C. Administer the medication sublingually and allow it to dissolve or absorb.

Which of the following is the MOST reliable method of estimating a patient's cardiac output? A. Listen to heart sounds with a stethoscope. B. Connect the patient to an electrocardiogram. C. Assess the heart rate and strength of the pulse. D. Determine the average diastolic blood pressure.

C. Assess the heart rate and strength of the pulse.

Which term refers to a heart rate below 60 beats per​ minute? A. Tachycardia B. Dysrhythmia C. Bradycardia D. Agonal

C. Bradycardia

Which of the following MOST accurately describes what the patient will experience during the postictal state that follows a seizure? A. hyperventilation and hypersalivation B. a rapidly improving level of consciousness C. confusion and fatigue D. a gradually decreasing level of consciousness

C. Confusion and fatigue

A 40-year-old man is in cardiac arrest. Your partner is performing CPR. You are attaching the AED when the patient's wife tells you that he has an automatic implanted cardiac defibrillator (AICD). The AED advises that a shock is indicated. What should you do? A. Avoid defibrillation as this will damage the patient's AICD. B. Contact medical control and request permission to defibrillate. C. Deliver the shock followed by immediate resumption of CPR. D. Continue CPR and transport the patient to the closest appropriate hospital.

C. Deliver the shock followed by immediate resumption of CPR.

The AED has just given a "no shock advised" message. Which of the following statements is true?

CPR must be resumed

Which of the following statements about infarction is​ correct? A. Ischemic pain from infarction is never relieved by nitroglycerin or morphine. B. The ischemic pain from infarction is the result of poor perfusion of the lungs as the heart falters. C. Infarction is commonly the result of a totally occluded coronary blood vessel. D. Infarction is typically the result of the​ patient's not taking prescribed cardiac medications.

C. Infarction is commonly the result of a totally occluded coronary blood vessel.

You are​ on-scene with a​ 48-year-old unresponsive male patient. Bystanders state he complained of chest pain and then suddenly collapsed. Vital signs are blood pressure​ 68/42, pulse​ 36, and respiratory rate 3. He is unresponsive to painful stimuli. After performing the primary assessment and treating all life​ threats, what is your next​ intervention? A. Administer​ high-concentration oxygen at 15 lpm by​ bag-valve mask. B. Insert an oral airway. C. Package the patient for rapid transport. D. Apply the AED.

C. Package the patient for rapid transport.

After administration of nitroglycerin to a patient with chest​ pain, which of the following is the side effect you most might expect to​ see? A. The patient develops hypertension. B. The patient becomes irritable. C. The patient develops a headache. D. The patient becomes dehydrated.

C. The patient develops a headache.

Which of the following situations could contraindicate the administration of​ aspirin? A. The patient is taking other medications. B. There is no water available with which to take the pills. C. The patient has a history of GI bleeding. D. The patient is having a heart attack.

C. The patient has a history of GI bleeding.

Which of the following statements regarding the administration of nitroglycerin tablets is true​? A. It takes 20 to 30 minutes for nitroglycerin to have an effect. B. If a​ patient's pulse rate changes following nitroglycerin​ administration, it indicates an allergic reaction. C. The patient may complain of a headache following administration. D. An increase in blood pressure should be expected.

C. The patient may complain of a headache following administration.

Which of the following is the​ EMT's role in caring for a patient presenting with symptoms of​ anxiety, irritability, a short​ temper, and chest pain with shortness of​ breath? A. Make an interpretation of the​ patient's cardiac rhythm. B. Provide automatic external defibrillation. C. Treat the patient as though he were having a heart problem. D. Determine the cause of the​ patient's chest pain.

C. Treat the patient as though he were having a heart problem.

You must reassess your patient within two minutes after administering​ nitroglycerin, as one of the side effects​ is: A. a sudden decrease in heart rate. B. numbness and tingling in the extremities. C. a decrease in blood pressure. D. a brief feeling of shortness of breath.

C. a decrease in blood pressure.

A 66-year-old female with a history of hypertension and diabetes presents with substernal chest pressure of 2 hours' duration. Her blood pressure is 140/90 mm Hg, her pulse is 100 beats/min and irregular, her respirations are 22 breaths/min, and her oxygen saturation is 92%. The patient does not have prescribed nitroglycerin, but her husband does. You should: A. obtain a SAMPLE history and contact medical control for advice. B. give her high-flow oxygen, attach the AED, and transport at once. C. administer oxygen, give her 324 mg of aspirin, and assess her further. D. give her one nitroglycerin and reassess her systolic blood pressure.

C. administer oxygen, give her 324 mg of aspirin, and assess her further.

A weakened area of an artery that balloons out and may​ rupture, causing catastrophic​ bleeding, is​ called: A. angina. B. angioplasty. C. an aneurysm. D. asystole.

C. an aneurysm.

When blood flow to a particular part of the brain is cut off by a blockage inside a blood vessel, the result is: A. a hemorrhagic stroke. B. atherosclerosis. C. an ischemic stroke. D. a cerebral embolism.

C. an ischemic stroke.

Febrile seizures: A. often result in permanent brain damage. B. are also referred to as petit mal seizures. C. are usually benign but should be evaluated. D. occur when a child's fever progressively rises.

C. are usually benign but should be evaluated.

Febrile seizures: A. often result in permanent brain damage. B. are also referred to as petit mal seizures. C. are usually benign but should be evaluated. D. occur when a child's fever progressively rises.

C. are usually benign but should be evaluated.

When assessing arm movement of a patient with a suspected stroke, you should: A. observe for approximately 5 minutes. B. expect to see one arm slowly drift down to the patient's side. C. ask the patient to close his or her eyes during the assessment. D. ask the patient to hold his or her arms up with the palms down.

C. ask the patient to close his or her eyes during the assessment.

THROMBOLYTIC THERAPY

Clot dissolvers may reverse stroke Sx if given within 2 to 3 hrs of Sx. Note this can't B used on hemorrhagic strokes.

A 49-year-old male presents with an acute onset of crushing chest pain and diaphoresis. You should: A. administer up to three doses of nitroglycerin. B. obtain vital signs and a SAMPLE history. C. assess the adequacy of his respirations. D. administer up to 324 mg of baby aspirin.

C. assess the adequacy of his respirations.

Law enforcement has summoned you to a nightclub, where a 22-year-old female was found unconscious in an adjacent alley. Your primary assessment reveals that her respirations are rapid and shallow and her pulse is rapid and weak. She is wearing a medical alert bracelet that identifies her as an epileptic. There is an empty bottle of vodka next to the patient. You should: A. place a bite block in her mouth in case she has a seizure and transport at once. B. apply oxygen via a nonrebreathing mask, place her on her left side, and transport. C. assist ventilations, perform a rapid exam, and prepare for immediate transport. D. apply oxygen via nonrebreathing mask and transport her for a blood-alcohol test.

C. assist ventilations, perform a rapid exam, and prepare for immediate transport.

The head and brain receive their supply of oxygenated blood from the: A. iliac arteries. B. brachial arteries. C. carotid arteries. D. subclavian arteries.

C. carotid arteries.

Interruption of cerebral blood flow may result from all of the following, EXCEPT: A. a thrombus. B. an embolism. C. cerebral vasodilation. D. an acute arterial rupture.

C. cerebral vasodilation.

Interruption of cerebral blood flow may result from all of the following, EXCEPT: A. a thrombus. B. an embolism. C. cerebral vasodilation. D. an acute arterial rupture.

C. cerebral vasodilation.

When would it be MOST appropriate for a patient to take his or her prescribed nitroglycerin? A. sharp chest pain that lasts longer than 10 to 15 minutes B. an acute onset of dizziness during a period of exertion C. chest pain that does not immediately subside with rest D. difficulty breathing that awakens the patient from sleep

C. chest pain that does not immediately subside with rest

Which of the following MOST accurately describes what the patient will experience during the postictal state that follows a seizure? A. hyperventilation and hypersalivation B. a rapidly improving level of consciousness C. confusion and fatigue D. a gradually decreasing level of consciousness

C. confusion and fatigue

The main legal risk in using the AED is: A. negligence on the part of the manufacturer. B. failure of the AED's internal computer chip. C. failing to deliver a shock when one is needed. D. not assessing for a pulse after a shock is delivered.

C. failing to deliver a shock when one is needed.

Risk factors for AMI that cannot be controlled include: A. excess stress. B. hyperglycemia. C. family history. D. lack of exercise.

C. family history.

You are dispatched to a convenience store for a patient who passed out. Upon arriving at the scene, you find two bystanders performing CPR on the patient, a 58-year-old male. Your initial action should be to: A. assess the effectiveness of the bystanders' CPR. B. quickly attach the AED and push the analyze button. C. have the bystanders stop CPR and assess the patient. D. request a paramedic unit and quickly attach the AED.

C. have the bystanders stop CPR and assess the patient.

The mental status of a patient who has experienced a generalized seizure: A. progressively worsens over a period of a few hours. B. is easily differentiated from that of acute hypoglycemia. C. is likely to improve over a period of 5 to 30 minutes. D. typically does not improve, even after several minutes.

C. is likely to improve over a period of 5 to 30 minutes.

The mental status of a patient who has experienced a typical seizure: A. progressively worsens over a period of a few hours. B. is easily differentiated from that of acute hypoglycemia. C. is likely to improve over a period of 5 to 30 minutes. D. typically does not improve, even after several minutes.

C. is likely to improve over a period of 5 to 30 minutes.

When afterload increases: A. the volume of venous blood that returns to the right atrium increases. B. the amount of resistance that the ventricle must beat against decreases. C. it becomes harder for the ventricle to push blood through the blood vessels. D. the blood pressure falls because of significant dilation of the blood vessels.

C. it becomes harder for the ventricle to push blood through the blood vessels.

Individuals with chronic alcoholism are predisposed to intracranial bleeding and hypoglycemia secondary to abnormalities in the: A. brain. B. kidneys. C. liver. D. pancreas.

C. liver.

Which of the following is NOT an assessment parameter included in the Cincinnati Prehospital Stroke Scale? A. speech B. arm drift C. memory D. facial droop

C. memory

You arrive at a local grocery store approximately 5 minutes after a 21-year-old female stopped seizing. She is confused and disoriented; she keeps asking you what happened and tells you that she is thirsty. Her brother, who witnessed the seizure, tells you that she takes valproate (Depakote) for her seizures, but has not taken it in a few days. He also tells you that she has diabetes. In addition to administering oxygen, you should: A. administer one tube of oral glucose and prepare for immediate transport. B. place her in the recovery position and transport her with lights and siren. C. monitor her airway and breathing status and assess her blood glucose level. D. give her small cups of water to drink and observe for further seizure activity.

C. monitor her airway and breathing status and assess her blood glucose level.

Angina pectoris occurs when: A. one or more coronary arteries suddenly spasm. B. myocardial oxygen supply exceeds the demand. C. myocardial oxygen demand exceeds supply. D. a coronary artery is totally occluded by plaque.

C. myocardial oxygen demand exceeds supply.

The EMT should use an AED on a child between 1 month and 8 years of age if: A. he or she is not breathing and has a weakly palpable pulse. B. his or her condition is rapidly progressing to cardiac arrest. C. pediatric pads and an energy-reducing device are available. D. special pads are used and the child has profound tachycardia.

C. pediatric pads and an energy-reducing device are available.

The left ventricle has the thickest walls because it: A. pumps blood to the lungs to be reoxygenated. B. uses less oxygen than other chambers of the heart. C. pumps blood into the aorta and systemic circulation. D. receives blood directly from the systemic circulation.

C. pumps blood into the aorta and systemic circulation.

A study into the effects of disease on blood composition would focus​ on: A. systolic and diastolic pressures. B. the peripheral and central pulses. C. red and white blood​ cells, platelets, and plasma. D. the​ arteries, veins,​ arterioles, venules, and capillaries.

C. red and white blood​ cells, platelets, and plasma.

A patient in cardiac arrest is wearing an external defibrillator vest, which is interfering with effective chest compressions. The EMT should A. leave the battery attached to the monitor and remove the vest. B. perform ventilations only and allow the vest device to defibrillate. C. remove the battery from the monitor and then remove the vest. D. remove the battery from the monitor and leave the vest in place

C. remove the battery from the monitor and then remove the vest.

Most AEDs are set up to adjust the voltage based on the impedance, which is the: A. direction that the electrical flow takes in the body. B. distance between the two AED pads on the chest. C. resistance of the body to the flow of electricity. D. actual amount of energy that the AED will deliver.

C. resistance of the body to the flow of electricity.

Which of the following veins is located inferior to the trunk? A. cephalic B. axillary C. saphenous D. subclavian

C. saphenous

During your treatment of a woman in cardiac arrest, you apply the AED, analyze her cardiac rhythm, and receive a "no shock advised" message. This indicates that: A. the AED has detected asystole. B. the AED detected patient motion. C. she is not in ventricular fibrillation. D. she has a pulse and does not need CPR.

C. she is not in ventricular fibrillation.

Hypoperfusion refers​ to: A. edema. B. dyspnea. C. shock. D. hypertension.

C. shock.

The electrical impulse generated by the heart originates in the: A. bundle of His. B. coronary sinus. C. sinoatrial (SA) node. D. atrioventricular (AV) node.

C. sinoatrial (SA) node.

In contrast to the sympathetic nervous system, the parasympathetic nervous system: A. prepares the body to handle stress. B. causes an increase in the heart rate. C. slows the heart and respiratory rates. D. dilates the blood vessels in the muscles.

C. slows the heart and respiratory rates.

When treating a patient with chest pain, you should assume that he or she is having an AMI because: A. angina usually occurs after an AMI. B. most patients with chest pain are experiencing an AMI. C. the cause of the pain cannot be diagnosed in the field. D. angina and AMI present identically.

C. the cause of the pain cannot be diagnosed in the field.

A dissecting aortic aneurysm occurs when: A. all layers of the aorta suddenly contract. B. a weakened area develops in the aortic wall. C. the inner layers of the aorta become separated. D. the aorta ruptures, resulting in profound bleeding.

C. the inner layers of the aorta become separated.

A transient ischemic attack (TIA) occurs when: A. medications are given to dissolve a cerebral blood clot. B. a small cerebral artery ruptures and causes minimal damage. C. the normal body processes destroy a clot in a cerebral artery. D. signs and symptoms resolve spontaneously within 48 hours.

C. the normal body processes destroy a clot in a cerebral artery.

A transient ischemic attack (TIA) occurs when: A. medications are given to dissolve a cerebral blood clot. B. a small cerebral artery ruptures and causes minimal damage. C. the normal body processes destroy a clot in a cerebral artery. D. signs and symptoms resolve spontaneously within 48 hours.

C. the normal body processes destroy a clot in a cerebral artery.

The left cerebral hemisphere controls: A. the right side of the face. B. heart rate and pupil reaction. C. the right side of the body. D. breathing and blood pressure.

C. the right side of the body.

A patient who is experiencing aphasia is: A. not able to swallow without choking. B. experiencing a right hemispheric stroke. C. unable to produce or understand speech. D. usually conscious but has slurred speech.

C. unable to produce or understand speech.

Los Angeles Prehospital Stroke Screen

CRITERION - Yes/Unknown/No 1. Age >45 y 2. History of seizures of epilepsy absent 3. Symptoms <24 h 4. At baseline, patient is not wheelchair-bound or bedridden 5. Blood glucose between 60 and 400 mg/dL 6. Obvious asymmetry (right versus left) in any of the following three exam categories (must be unilateral): Equal/Right Weak/Left Weak - Facial smile/grimace *Droop* - Grip *Weak grip/No grip* - Arm strength *Drifts down/Falls rapidly* If all criteria 1-6 are marked yes, the probability of a stroke is 97%.

Most common inhaled drug

Carbon monoxide

Sudden Death

Cardiac Arrest that occurs within 2 hours of the onset of symptoms. -Patient may have no prior symptoms A cardiac arrest that occurs within 2 hours of the onset of symptoms. The patient may have no prior symptoms of coronary artery disease

To standardize the measurement of cardiac output in individuals of different sizes, the value for cardiac output is divided by the persons body surface area. The result is known as?

Cardiac Index (CI): used to describe flow output

In which of the following ways does cardiac arrest in children differ from cardiac arrest in adults?

Cardiac arrest in children is more likely to be due to respiratory failure.

"sudden death"

Cardiac arrest that occurs within 2 hours of the onset of symptoms; patient may or may not have prior symptoms of CAD

Which of the following is the general term used to refer to a problem with the heart?

Cardiac compromise

Delivery of an electrical shock to restore a normal heart rhythm most directly affects which of the​ following? A. Cardiac conduction system B. Peripheral pulses C. Blood circulation D. Blood pressure

Cardiac conduction system

The amount of blood pumped out of the left ventricle in 1 minute is the definition of...

Cardiac output

pulmonary veins

Carries oxygenated blood to the heart the ___________ __________________________.

Tension headaches

Caused by muscle contractions in the head and neck and are attributed to stress. Patients usually describe the pain as squeezing, dull, or an ache.

Sinus headaches

Caused by pressure that is the result of fluid accumulation in the sinus cavities.

CAUSES OF SINUS HEADACHES

Caused by the accumulation of fluid in the sinus cavities. Patients may also have cold like Sx such as nasal congestion.

SPECIAL CONSIDERATION FOR PEDIATRICS

Causes of altered mental status in children (strokes, seizures, high / low blood glucose) Meningitis - causes of altered mental status Seizures - febrile seizures

infarction

Cell death is known as

Interruption of cerebral blood flow may results from all of the following, EXCEPT:

Cerebral vasodilation.

The three major parts of the brain are the:

Cerebrum, cerebellum, and brain stem.

fibrillation

Chaotic beating of the heart

What is the best known symptom of a heart problem?

Chest pain

What are the indications for administration of aspirin?

Chest pain Not allergic to aspirin No history of aspirin Not already taking blood thinners No other contraindications to aspirin Is able to swallow Medical direction authorizes administration

What are the indications for nitroglycerin?

Chest pain Patient has history of cardiac problems Patient has nitro prescription BP is 90 or more Medical direction authorizes use

You suspect that your patient is suffering from angina pectoris. What signs or symptoms would you expect to see with this condition?

Chest pain that is relieved with nitroglycerin

what is the most common symptom of a heart problem?

Chest pain.

You are treating a patient who is exhibiting slurred speech, facial droop, and an inability to move his left arm. Which neurologic examination emphasizes these possible stroke signs?

Cincinnati Prehospital Stroke Scale

You respond to the scene of a 56-year-old obese female complaining of respiratory distress. She states that she has been feeling weak and a "little sick" for the past two days but the respiratory distress has been getting progressively worse for the past several hours. She states she has "heart problems," suffers from high blood pressure, and takes a "water pill." She is afebrile and has coarse crackles (rales) bilaterally. What is most likely the cause of her respiratory distress?

Congestive heart failure

Which heart condition/emergency causes edema?

Congestive heart failure (CHF)

You are on the scene of a 65-year-old female patient in cardiac arrest. CPR is in progress and the AED has been applied. The AED advises shock. After defibrillating the patient, what is your next intervention?

Continue CPR.

You are on the scene of a 65-year-old female patient in cardiac arrest. CPR is in progress and the AED has been applied. The AED does not advise shock. What is your next intervention?

Continue CPR.

Which of the following should NOT be done during defibrillation?

Continued ventilation during the analysis phase to prevent hypoxia

Front part of the cerebrum

Controls emotion and thought.

Middle part of the cerebrum

Controls sensation and movement.

bi-cuspid or mitral valve

Controls the blood flow between the left atrium and left ventricle, has two names

BRAIN STEM

Controls the most basic functions of the body such as breathing, BP, swallowing & pupil constriction.

The brain stem

Controls the most basic functions of the body, such as breathing, blood pressure, swallowing, and pupil construction.

What does CAD mean?

Coronary artery disease

What is usually the underlying reason for an AMI?

Coronary artery disease

Which of the following is the name of the condition in which fatty deposits form in the inner lining of the arteries?

Coronary artery disease

What happens in an angina attack?

Coronary artery disease has narrowed the arteries that supply the heart. During times of stress, the heart works harder and the portion of the heart supplied by the narrowed artery is deprived of oxygen, resulting in chest pain.

CAD?

Coronary artery disease: disease that affects the arteries in the heart.

Which of the following is the result of a portion of the heart muscle dying due to a lack of​ oxygen? A. Heart failure B. Angina pectoris C. Cardiac arrest D. Myocardial infarction

D. Myocardial infarction

Which of the following may be a symptom of a problem with the​ heart? A. ​Nausea, with or without vomiting B. Mild chest discomfort C. ​Severe, crushing pain in the chest D. All of the above

D. All of the above

Which of the following skills is typically NOT used to manage the airway and breathing of a patient in cardiac​ arrest? A. Opening the airway manually B. Suctioning C. Insertion of an oropharyngeal airway D. Application of oxygen via nasal cannula

D. Application of oxygen via nasal cannula

Your patient is a​ 44-year-old female who has collapsed while jogging. She has been unresponsive for four to five minutes by the time you arrive. Her husband appears to be performing​ high-quality CPR. Which of the following should be your first​ action? A. Insert an oropharyngeal airway and begin ventilations. B. Load the patient into the ambulance for further assessment. C. Stop CPR and check for a pulse. D. Apply the AED.

D. Apply the AED.

In which of the following ways does cardiac arrest in children differ from cardiac arrest in​ adults? A. Ventricular fibrillation is not common in adults. B. Ventricular fibrillation is common in children. C. Cardiac arrest in adults is more likely to be due to respiratory failure. D. Cardiac arrest in children is more likely to be due to respiratory failure.

D. Cardiac arrest in children is more likely to be due to respiratory failure.

Which of the following is the name of the condition in which fatty deposits form in the inner lining of the arteries of the​ heart? A. Arteriosclerosis B. Coronary thrombosis C. Aneurysm D. Coronary artery disease

D. Coronary artery disease (arteriosclerosis is the thickening/hardening/loss of elasticity of artery walls)

Which of the following is an expected side effect of​ aspirin? A. Hypertension B. Increased clotting C. Fever D. Heartburn

D. Heartburn

Contraindications for the administration of nitroglycerin include a​ patient: A. who is 22 years of age. B. who is suffering from hypertension. C. who has already taken one dose of nitroglycerin without relief. D. who recently took Viagra.

D. who recently took Viagra.

Which of the following statements about infarction is​ CORRECT? A. Ischemic pain from infarction is never relieved by nitroglycerin or morphine. B. Infarction is typically the result of the patient not taking prescribed cardiac medications. C. The ischemic pain from infarction is the result of poor perfusion of the lungs as the heart falters. D. Infarction is commonly the result of a totally occluded coronary blood vessel.

D. Infarction is commonly the result of a totally occluded coronary blood vessel.

You arrive​ on-scene of a​ 56-year-old female patient in cardiac arrest. What is the first airway you should​ administer? A. Combitube B. Nasopharyngeal airway​ (NPA) C. ​Pharyngeal-Tracheal Lumen airway D. Oropharyngeal airway

D. Oropharyngeal airway

You are on the scene of a​ 68-year-old male patient complaining of severe chest pain for the last 20 minutes. He has a previous history of myocardial infarction and states that it feels​ "just like the last​ time." You have applied oxygen and assisted him in administering aspirin and nitroglycerin with no reduction in the chest pain. Your nearest facility is five minutes​ away, a Level III Trauma Center is 10 minutes​ away, and a hospital with cardiac catheterization capabilities is 20 minutes away. The patient is requesting to be transported to his​ cardiologist's hospital, which is 30 minutes away. Which hospital is the best​ destination? A. The nearest facility B. The hospital the patient requests C. The trauma center D. The nearest hospital with cardiac catheterization

D. The nearest hospital with cardiac catheterization

Which of the following is a contraindication for use of the​ AED? A. The patient is under 8 years of age. B. The patient is hypothermic. C. The patient has been down for over 10 minutes. D. The patient is in contact with metal.

D. The patient is in contact with metal.

A paramedic unit has arrived on the scene of a cardiac arrest where you have been performing CPR and using an AED. The crew has started an IV and inserted an endotracheal tube. How will this change how you do your​ job? A. The AED should be immediately removed. B. You will now shock once every minute because drugs may be given. C. You no longer need to do compressions because the paramedics have their monitor. D. Ventilations may be delivered asynchronously from chest compressions now.

D. Ventilations may be delivered asynchronously from chest compressions now.

An​ 80-year-old male is presenting with pedal​ edema, JVD, and dyspnea. You should​ suspect: A. ​multi-system trauma. B. anaphylaxis. C. pulmonary embolism. D. congestive heart failure.

D. congestive heart failure.

A patient with a ventricular assist device​ will: A. have palpably high blood pressure. B. have a palpably slow pulse. C. not have palpably strong peripheral pulses. D. not have a palpable pulse or blood pressure.

D. not have a palpable pulse or blood pressure.

A​ 54-year-old male is found pulseless and apneic on the sofa. The scene is safe. You should​ first: A. apply oxygen. B. call medical control. C. call for ALS. D. start CPR and retrieve an AED.

D. start CPR and retrieve an AED.

​Auto-Pulse is​ designed: A. to start only if an advanced airway is in place. B. to start only if​ bag-valve mask ventilation is applied. C. to be started automatically. D. to be started manually.

D. to be started manually.

A patient tells you that he has a left ventricular assist device (LVAD). Which of the following conditions should you suspect that he has experienced? A. Obstructive lung disease B. Uncontrolled hypertension C. Thoracic aortic aneurysm D. Acute myocardial infarction

D. Acute myocardial infarction

Which of the following may be a sign or symptom of a problem with the​ heart? A. Nausea, with or without vomiting B. Chest discomfort C. Fainting or near fainting D. All of the above

D. All of the above

You suspect that your patient is suffering from angina pectoris. What signs or symptoms would you expect to see with this​ condition? A. Chest pain that is not relieved with nitroglycerin B. Chest pain that radiates to the leg C. Chest pain that is not relieved with rest D. Chest pain that is relieved with nitroglycerin

D. Chest pain that is relieved with nitroglycerin

Which of the following is a contraindication for​ aspirin? A. Liver disease B. Cardiovascular disease C. Having taken nitroglycerin D. Inability to swallow without endangering airway

D. Inability to swallow without endangering airway

Which of the following statements regarding angina pectoris is true​? A. It results in death of a smaller portion of myocardium than does a heart attack. B. It is generally relieved by​ over-the-counter medications. C. It generally lasts 30 to 60 minutes. D. It can be brought on by exertion or stress.

D. It can be brought on by exertion or stress.

Which of the following statements regarding the pain associated with AMI is correct? A. It is often described by the patient as a sharp feeling. B. It often fluctuates in intensity when the patient breathes. C. Nitroglycerin usually resolves the pain within 30 minutes. D. It can occur during exertion or when the patient is at rest.

D. It can occur during exertion or when the patient is at rest.

Which of the following is the beneficial action of nitroglycerin in some cardiac​ emergencies? A. It dilates only the coronary arteries. B. It increases the strength with which the ventricles contract. C. It slows down the heart. D. It relaxes blood vessels throughout the body.

D. It relaxes blood vessels throughout the body.

Which of the following is typical of angina​ pectoris? A. Lasts 10 to 30 seconds B. Does not respond to nitroglycerin C. Does not present following stress D. May subside with rest

D. May subside with rest

Which of the following statements regarding nitroglycerin is correct? A. The potency of nitroglycerin is increased when exposed to light. B. A maximum of five nitroglycerin doses should be given to a patient. C. Nitroglycerin should be administered between the cheek and gum. D. Nitroglycerin usually relieves anginal chest pain within 5 minutes.

D. Nitroglycerin usually relieves anginal chest pain within 5 minutes.

Which of the following terms refers to swelling of the lower extremities seen in many cardiac​ patients? A. Presacral swelling B. Crackles C. Congestive heart failure D. Pedal edema

D. Pedal edema

Which of the following statements about angina pectoris is​ true? A. Angina pectoris is the leading cause of sudden death. B. The symptoms of angina pectoris do not resolve on their own. C. Pain from angina pectoris usually begins when the patient is at rest. D. Rest or nitroglycerin usually relieves angina pectoris.

D. Rest or nitroglycerin usually relieves angina pectoris.

Chest pain due to angina or myocardial infarction involves a blockage in what part of the cardiovascular​ system? A. The venules B. The veins C. The capillaries D. The arteries

D. The arteries

Which of the following symptoms would lead the EMT to believe that a patient's headache is caused by sinus congestion? A. The headache began suddenly B. There is associated neck stiffness C. There is numbness in the extremities D. The pain is worse when bending over

D. The pain is worse when bending over

Which of the following best describes the pain associated with acute coronary​ syndrome? A. The pain is a pressure in the chest. B. The pain is only a mild discomfort. C. The pain will go away after a few hours. D. The pain may be chest​ pressure, mild​ discomfort, or not present at all.

D. The pain may be chest​ pressure, mild​ discomfort, or not present at all.

Common signs and symptoms of a hypertensive emergency include: A. pallor, cool skin, and a temporary loss of hearing. B. syncope, a weak pulse, and bleeding from the ears. C. tachycardia, pain behind the eyes, and weakness. D. a bounding pulse, a severe headache, and dizziness.

D. a bounding pulse, a severe headache, and dizziness.

A 58-year-old male presents with confusion, right-sided weakness, and slurred speech. His wife is present and is very upset. As your partner is applying oxygen, it is MOST important for you to: A. list all of the patient's current medications. B. obtain a complete set of baseline vital signs. C. administer glucose to rule out hypoglycemia. D. ask his wife when she noticed the symptoms.

D. ask his wife when she noticed the symptoms.

A 49-year-old male presents with an acute onset of crushing chest pain and diaphoresis. You should: A. administer up to 324 mg of baby aspirin. B. administer up to three doses of nitroglycerin. C. obtain vital signs and a SAMPLE history. D. assess the adequacy of his respirations.

D. assess the adequacy of his respirations.

Narrowing of the coronary arteries due to a buildup of fatty deposits is called: A. angina pectoris. B. arteriosclerosis. C. acute ischemia. D. atherosclerosis.

D. atherosclerosis.

The ability of cardiac muscle cells to contract spontaneously without a stimulus from a nerve source is called: A. excitability. B. contractility. C. impulsivity. D. automaticity.

D. automaticity.

The electrical stimulus that originates in the heart's primary pacemaker is controlled by impulses from the brain that arrive by way of the: A. parietal lobe. B. pons and medulla. C. somatic nervous system. D. autonomic nervous system.

D. autonomic nervous system.

The posterior tibial pulse can be palpated: A. on the dorsum of the foot. B. above the lateral malleolus. C. in the fossa behind the knee. D. behind the medial malleolus.

D. behind the medial malleolus.

The most basic functions of the body, such as breathing, blood pressure, and swallowing, are controlled by the: A. cerebellum. B. cerebrum. C. cerebral cortex. D. brain stem.

D. brain stem.

The three major parts of the brain are the: A. cerebellum, medulla, and occiput. B. brain stem, midbrain, and spinal cord. C. midbrain, cerebellum, and spinal cord. D. cerebrum, cerebellum, and brain stem.

D. cerebrum, cerebellum, and brain stem.

The three major parts of the brain are the: A. cerebellum, medulla, and occiput. B. brain stem, midbrain, and spinal cord. C. midbrain, cerebellum, and spinal cord. D. cerebrum, cerebellum, and brain stem.

D. cerebrum, cerebellum, and brain stem.

The myocardium receives oxygenated blood from the __________, which originate(s) from the __________. A. coronary sinus, vena cava B. aorta, inferior vena cava C. vena cava, coronary veins D. coronary arteries, aorta

D. coronary arteries, aorta

A percutaneous transluminal coronary angioplasty (PTCA) restores blood flow to the ischemic myocardium by: A. scraping fatty deposits off of the lumen of the coronary artery. B. bypassing the coronary artery with a vessel from the chest or leg. C. placing a stent inside the coronary artery to keep it from narrowing. D. dilating the affected coronary artery with a small inflatable balloon.

D. dilating the affected coronary artery with a small inflatable balloon.

SEIZURE

Generalized, uncoordinated muscular activity associated w a LOC;a convulsion.

A 29-year-old male complains of a severe headache and nausea that has gradually worsened over the past 12 hours. He is conscious, alert, and oriented and tells you that his physician diagnosed him with migraine headaches. He further tells you that he has taken numerous different medications, but none of them seem to help. His blood pressure is 132/74 mm Hg, his pulse is 110 beats/min and strong, and his respirations are 20 breaths/min and adequate. In addition to high-flow oxygen, further treatment should include: A. assisting him with his migraine medication and transporting without lights and siren. B. placing him in a supine position and transporting with lights and siren to a stroke center. C. applying warm compresses to the back of his neck and transporting with lights and siren. D. dimming the lights in the back of the ambulance and transporting without lights and siren.

D. dimming the lights in the back of the ambulance and transporting without lights and siren.

During the primary assessment of a semiconscious 70-year-old female, you should: A. insert a nasopharyngeal airway and assist ventilations. B. immediately determine the patient's blood glucose level. C. ask family members if the patient has a history of stroke. D. ensure a patent airway and support ventilation as needed.

D. ensure a patent airway and support ventilation as needed.

Prior to defibrillating a patient with an AED, it is MOST important that you: A. properly position the defibrillation pads. B. perform up to 5 minutes of effective CPR. C. confirm that the patient is in cardiac arrest. D. ensure that no one is touching the patient.

D. ensure that no one is touching the patient.

Which of the following is of LEAST pertinence when obtaining medical history information from a patient complaining of chest discomfort? A. history of cigarette smoking B. history of previous heart attack C. presence of personal risk factors D. family history of hypertension

D. family history of hypertension

A patient who is possibly experiencing a stroke is NOT eligible for thrombolytic (fibrinolytic) therapy if he or she: A. has had a prior heart attack. B. is older than 60 years of age. C. has a GCS score that is less than 8. D. has bleeding within the brain.

D. has bleeding within the brain.

You suspect that your patient is experiencing acute coronary syndrome. Your hospital destination should be determined​ by: A. the​ patient's insurance. B. patient preference. C. traffic conditions. D. local protocols.

D. local protocols.

You arrive at the residence of a 33-year-old woman who is experiencing a generalized (tonic-clonic) seizure. She has a small amount of vomitus draining from the side of her mouth. After protecting her from further injury, you should: A. place a bite block in between her teeth, apply high-flow oxygen via a nonrebreathing mask, and consider inserting a nasopharyngeal airway. B. wait for the seizure to stop, manually open her airway, insert an oropharyngeal airway, and assess her oxygen saturation with the pulse oximeter. C. restrain her extremities to prevent her from injuring herself, suction her airway to remove the vomitus, and assist her ventilations with a bag-mask device. D. maintain her airway with manual head positioning, suction her airway to remove the vomitus, insert a nasopharyngeal airway, and administer high-flow oxygen.

D. maintain her airway with manual head positioning, suction her airway to remove the vomitus, insert a nasopharyngeal airway, and administer high-flow oxygen.

A patient with an altered mental status is: A. completely unresponsive to all forms of stimuli. B. typically alert but is confused as to preceding events. C. usually able to be aroused with a painful stimulus. D. not thinking clearly or is incapable of being aroused.

D. not thinking clearly or is incapable of being aroused.

When preparing to obtain a 12-lead ECG, the "LL" and "RL" electrodes should be placed: A. anywhere on the arms. B. on either side of the chest. C. on the lower abdomen. D. on the thighs or ankles.

D. on the thighs or ankles.

Common signs and symptoms of AMI include all of the following, EXCEPT: A. irregular heartbeat. B. sudden unexplained sweating. C. shortness of breath or dyspnea. D. pain exacerbated by breathing.

D. pain exacerbated by breathing.

A 67-year-old female presents with difficulty breathing and chest discomfort that awakened her from her sleep. She states that she has congestive heart failure, has had two previous heart attacks, and has prescribed nitroglycerin. She is conscious and alert with adequate breathing. Her blood pressure is 94/64 mm Hg and her heart rate is 120 beats/min. Treatment for this patient includes: A. nitroglycerin for her chest pain. B. ventilations with a bag-mask device. C. oxygen at 4 L/min via nasal cannula. D. placing her in an upright position.

D. placing her in an upright position.

Status epilepticus is characterized by: A. generalized seizures that last less than 5 minutes. B. an absence seizure that is not preceded by an aura. C. profound tachycardia and total muscle flaccidity. D. prolonged seizures without a return of consciousness.

D. prolonged seizures without a return of consciousness.

Which of the following blood vessels transports oxygenated blood? A. superior vena cava B. pulmonary arteries C. inferior vena cava D. pulmonary veins

D. pulmonary veins

The primary job of the heart is​ to: A. remove wastes from the blood. B. store blood. C. oxygenate the blood. D. pump blood.

D. pump blood.

The AED has delivered a shock to an elderly male in cardiac arrest. Following 2 minutes of CPR, you reanalyze the patient's cardiac rhythm and receive a "no shock advised" message. After further resuscitation, you restore a palpable carotid pulse. Your next action should be to: A. obtain a blood pressure and apply the pulse oximeter. B. place him in the recovery position and apply oxygen. C. transport at once and reanalyze his rhythm en route. D. reassess airway and breathing and treat accordingly.

D. reassess airway and breathing and treat accordingly.

Which of the following conditions would MOST likely affect the entire brain? A. blocked cerebral artery in the frontal lobe B. reduced blood supply to the left hemisphere C. ruptured cerebral artery in the occipital lobe D. respiratory failure or cardiopulmonary arrest

D. respiratory failure or cardiopulmonary arrest

Defibrillator pads are placed on the patient's chest with one pad to the: A. left of the upper sternum and the other pad just to the right of the left nipple. B. right of the upper sternum and the other pad just to the right of the right nipple. C. left of the upper sternum and the other pad just to the right and below the left nipple. D. right of the upper sternum and the other pad just to the left and below the left nipple.

D. right of the upper sternum and the other pad just to the left and below the left nipple.

The electrical impulse generated by the heart originates in the: A. atrioventricular node. B. coronary sinus. C. bundle of His. D. sinoatrial node.

D. sinoatrial node.

Hypoglycemia can mimic conditions such as: A. cystic fibrosis. B. myocardial infarction. C. high fever. D. stroke.

D. stroke.

Which of the following clinical signs is MOST suggestive of a ruptured cerebral artery? A. unilateral hemiparesis B. confusion and weakness C. nasal discharge of blood D. sudden, severe headache

D. sudden, severe headache

Ventricular tachycardia causes hypotension because: A. the volume of blood returning to the atria increases. B. the right ventricle does not adequately pump blood. C. blood backs up into the lungs and causes congestion. D. the left ventricle does not adequately fill with blood.

D. the left ventricle does not adequately fill with blood.

Cardiac output may decrease if the heart beats too rapidly because: A. a rapid heart beat causes a decrease in the strength of cardiac contractions. B. the volume of blood that returns to the heart is not sufficient with fast heart rates. C. as the heart rate increases, more blood is pumped from the ventricles than the atria. D. there is not enough time in between contractions for the heart to refill completely.

D. there is not enough time in between contractions for the heart to refill completely.

Successful treatment of a stroke depends on whether: A. surgical intervention is performed to remove obstructive clots. B. medications are given to restore the function of infarcted cells. C. the stroke occurs within the left or right hemisphere of the brain. D. thrombolytic therapy is given within 3 hours after symptoms began.

D. thrombolytic therapy is given within 3 hours after symptoms began.

The principal clinical difference between a stroke and hypoglycemia is that patients with hypoglycemia: A. always take oral medications to maintain normal blood glucose levels. B. do not present with slurred speech or weakness to one side of the body. C. are typically alert and attempt to communicate with health care providers. D. usually have an altered mental status or decreased level of consciousness.

D. usually have an altered mental status or decreased level of consciousness.

Which of the following cardiac arrhythmias has the greatest chance of deteriorating into a pulseless rhythm? A. sinus tachycardia B. sinus bradycardia C. extra ventricular beats D. ventricular tachycardia

D. ventricular tachycardia

Intermittent positive-pressure breathing (IPPB) with positive end-expiratory pressure (PEEP) has what effect on afterload and preload?

Decreases afterload (resistance) Decreases preload

Which of the following will provide a cardiac arrest patient the greatest chance of​ survival? A. Defibrillation within two minutes B. Early advanced cardiac life support C. Advanced airway management D. Rapid transport

Defibrillation within two minutes

Which of the following is the basis for applying an AED only to patients who are not breathing and who do NOT have a pulse?

Defibrillators recognize ventricular tachycardia, which may be accompanied by a pulse.

A 40-year-old man is in cardiac arrest. Your partner is performing CPR. You are attaching the AED when the patient's wife tells you that he has an implanted pacemaker. The AED advises that a shock is indicated. What should you do?

Deliver the shock followed by immediate resumption of CPR.

What does dyspnea mean?

Difficulty breathing

Patients with a heart problems may complain of..

Difficulty breathing, mild chest discomfort, and pain in the center of the chest

Coronary Artery Disease

Diseases that affect the arteries of the heart

Dysrhythmia

Disturbance in heart rate and rhythm (including irregular and absent heart rhythm)

nitroglycerin

Drug in EMT scope that causes vasodilation. A medication that dilates the blood vessels.

Narcotics

Drugs that affect the nervous system

A patient whose speech is slurred and difficult to understand is experiencing:

Dysarthria.

Which intervention is proven to be the most effective is obtaining a return of spontaneous circulation​ (ROSC) in a cardiac arrest​ patient?

Early Defibrillation

What BEST describes the​ EMT's intended role in the ideal provision of emergency cardiac​ care?

Early defibrillation

Which intervention is proven to be the most effective is obtaining a return of spontaneous circulation (ROSC) in a cardiac arrest patient?

Early defibrillation

Which of the following BEST describes the EMT's intended role in the ideal provision of emergency cardiac care?

Early defibrillation

Which of the following would NOT be considered an element of​ post-cardiac arrest​ care? A. Early defibrillation B. Inducing hypothermia C. Maintaining adequate oxygenation D. ​12-lead ECG

Early defibrillation

Headache, vomiting, altered mental status, and seizures.

Early signs of increased intracranial pressure include:

right sided CHF

Edema of the lower legs might indicate

CAUSES OF SEIZURES

Epileptic (congenital) Structural (tumor, infection, scar tissue, head trauma, stroke) Metabolic (hypoxia, abnormal blood chemical values, hypoglycemia, poisoning, drug overdose, sudden withdrawal from alcohol, medications Febrile - Sudden high fever

Congestive heart failure

Excessive amount of fluid built up in the lungs or organs

CINCINNATI PREHOSPITAL STROKE SCALE

Facial Droop (Ask patient 2 smile) Normal - Both sides of face move equally well. Abnormal - 1 side of the face does not move as well as the other. Arm Drift (Ask patient 2 close eyes & hold both arms out w palms up) Normal - Both arms move the same, or both arms do not move. Abnormal - 1 arm does not move, or 1 arm drifts down compared w the other side. Speech (Ask patient 2 say "The sky is blue in Cincinnati") Normal-patient uses correct words w no slurring abnormal-patient slurs words, uses inappropriate words, or is unable 2 speak.

Pedal edme

Fluid built up in feet and ankles causing swelling.

Pulmonary edmea

Fluid built up in the lungs

SIGNS OF INCREASED INTRACRANIAL PRESSURE

Headache, vomiting, AMS & seizures. Increasing intracranial pressure can b from hemorrhagic stroke or from head trauma.

Cardiac output is a product of which two variables?

Heart rate (HR) and Stroke volume (SV)

The aortic valve

Heart valve between the left ventricle and the aorta.

Pulseless electrical activity

Hearts electrical Rhythm is normal yet mechanical pumping activity fails causing cardiac arrest.

Ventricular fibrillation

Hearts electrical impulses are disorganized

hypertensive?

High blood pressure.

You are treating a male patient with chest pain caused by the complete occlusion of one of the coronary arteries. What would you most likely expect with this patient?

His pain will not be alleviated with any of the above medications.

When AED delivers shock inappropriately what is the cause?

Human error

The MOST significant risk factor for a hemorrhagic stroke is:

Hypertension.

what mimics a stroke and also causes a seizure?

Hypoglycemia

Which of the following conditions would be the LEAST likely to mimic the signs and symptoms of a stroke?

Hypovolemia.

Common causes of Metabolic seizures

Hypoxia Abnormal blood chemical values Hypoglycemia Poisoning Drug overdose Sudden withdrawal from alcohol or medications

3-Item Stroke Severity Scale (LAG)

ITEM/CRITERIA/SCORE Level of consciousness: - Normal [0] - Mild dysfunction [1] - Severe dysfunction (unconscious) [2] Arm drift (hemiparesis): - Normal function [0] - Mild dysfunction [1] - Severe dysfunction (flaccid) [2] Gaze: - Normal gaze (follows pen/finger to left and right sides) [0] - Mild dysfunction [1] - Severe dysfunction (fixed gaze) [2] Score (total): >4 indicates stroke is likely

What are the contraindications for an AED?

If the lads will not fit without touching If the patient is wet or in water If the patient is touching metal If the patient is wearing a nitroglycerine patch

Which of the following BEST describes the purpose of a continuous quality improvement program for AED?

Improving patient outcomes in the community

INCIDENCE OF SEIZURE

Incidence of seizures is high representing 30% of 911 calls. Estimated 4 million people have seizures.

Which of the following statements about infarction is​ CORRECT? A. Infarction is typically the result of the patient not taking prescribed cardiac medications. B. Ischemic pain from infarction is never relieved by nitroglycerin or morphine. C. Infarction is commonly the result of a totally occluded coronary blood vessel. D. The ischemic pain from infarction is the result of poor perfusion of the lungs as the heart falters.

Infarction is commonly the result of a totally occluded coronary blood vessel.

3 types of poisons

Inhaled, ingested, absorbed

You are on the scene of a 70-year-old female patient in cardiac arrest. CPR is in progress and the AED has been applied. After defibrillating the patient with an AED, you get return of spontaneous circulation but the patient is still unresponsive and in respiratory arrest. What is your next intervention?

Insert a Combitube.

Ago almost breathing

Irregular, gasping breathes that precede apnea or death.

What is agonal breathing?

Irregular, gasping breaths that precede apnea and death

What is ACS?

Is a blanket term that refers to any time the heart may not be getting enough oxyegn

What is cardiac compromise?

Is a blanket term that refers to any time the heart may not be getting enough oxyegn

Patient W Seizure Refuses Transport. Which Questions Do U Ask?

Is the patient awake & completely oriented after the seizure (GCS 15) Does ur assessment show no evidence of trauma or complications from the seizure? Has ur patient ever had a seizure before? Was this seizure the "usual" seizure in every way (length, activity, recovery)? Is the patient currently being treated w medication & receiving regular evaluations by a physician?

What best describes preload?

Is the stretch on the ventricular muscle fibers before contraction. Is created by the EDV Starling's law (the greater the stretch on the resting ventricle, the greater the contraction up to a critical limit)

What happens when blood flow to a particular part of the brain is cut off by a blockage, resulting in tissue damage?

Ischemic stroke

Which of the following statements regarding angina pectoris is true?

It can be brought on by exertion or stress.

Which of the following statements regarding the pain associated with AMI is correct?

It can occur during exertion or when the patient is at rest.

Which of the following is the beneficial action of nitroglycerin in some cardiac emergencies?

It dilates blood vessels throughout the body.

- Time of onset of the signs and symptoms - Score on the Glascow Coma Scale - Results of a stroke assessment tool (Cincinnati, Los Angeles, LAG, or FAST) - Changes noted on reassessment

Key information to document for a patient who may have had a stroke:

Damage to the _______ Ventricle causes the backup of blood into the lungs.

Left

The filling pressure of the left heart also is known as the _________ pressure. The PCWP is used to measure the left ventricular filling pressure.

Left Atrial Pressure Pulmonary Capillary Wedge Pressure (PCWP)

You are dispatched to a home where you find a 70-year-old man. He is exhibiting facial drooping and, when you ask him to tell you what day it is, he says "January." Which part of the brain is most likely to have been affected?

Left hemisphere

Which of the following heart chambers pumps oxygenated blood to the body?

Left ventricle

CEREBRUM

Located above the cerebellum it is divided into right & left hemispheres & each hemisphere controls the opposite side of the body. Front controls emotion & thought, middle controls touch & movement & the back controls sight. The left / middle of the cerebrum controls speech.

The cerebrum

Located above the cerebellum, is divided down the middle into the right and left cerebral hemispheres. Each hemisphere controls activities on the opposite side of the body.

What happens in asystole?

The heart has ceased generating electrical impulses altogether

CEREBELLUM

Located just behind the brain stem the cerebellum controls body coordination. It is responsible for coordinating complex tasks that involve many muscles such as walking, writing & playing piano.

The cerebellum

Located just behind the brain stem, which controls muscle and body coordination; Is responsible for coordinating complex tasks that involve many muscles

INCONTINENCE

Loss of bowel / bladder control;may B the result of a generalized seizure.

Incontinence

Loss of bowel and/or bladder control; may be the result of a generalized seizure.

Which of the following is NOT a typical indication of congestive heart failure?

Low blood pressure

hypotensive?

Low blood pressure.

________ affect(s) the entire brain, often causing anxiety, restlessness, and confusion.

Low oxygen levels in the bloodstream

When the Fick method is used to calculate cardiac output, if C(a-v)O2 is greater than expected and VO2 is normal, cardiac output probably will be?

Lower Higher C(a-v)O2 valves indicate more O2 was removed, as when the following occurs: Blood flow is slow (CO) or, Tissue extraction of O2 is high (O2 consumption)

The cardiac output of the right ventricle is the same as the cardiac out put of the left ventricle. Compared with the left ventricle, the cardiac work index of the right ventricle is?

Lower Right Ventricle: has to eject blood against pulmonary pressure [resistance] Left Ventricle: Must eject against the mean aortic pressure (MAP), which is 6x the mean pulmonary artery pressure (MPAP), Cardiac work is much higher for left ventricle.

What are the side effects of nitroglycerine?

Lower blood pressure Headache Pulse rate changes

superior and inferior vena cava

Major veins that return the blood directly to the heart

Absence seizure (formerly called petit mal)

May last just for seconds, after which the patient fully recovers with only a brief lapse of memory of the event.

cardiac output

Measure of the blood circulated by the heart in 1 minute. HR x stroke volume.

What are fibrinolytics?

Medications that dissolve clots. The faster after the discovery of the clot the more effective they are.

- Levetiracetam (Keppra) - Phenytoin (Dilantin) - Phenobarbital - Carbamazepine (Tegretol) - Valproate (Depakote) - Topiramate (Topamax) - Clonazepam (Klonopin)

Medications used most often to treat seizures include the following:

When using a mechanical CPR device, you must:

Minimize interruptions in chest compressions

100mm/hg systolic

Minimum BP to administer nitro

80mm/hg

Minimum BP to feel a radial pulse

Which of the following is the result of a portion of the heart muscle dying due to a lack of oxygen?

Myocardial infarction

What may cause a decrease in ventricular compliance?

Myocardial ischemia and infarction Hemorrhagic and septic shock Pericardial effusions Right ventricular dilation and overload PEEP CPAP Inotropic drugs

What are the side effects of aspirin?

Nausea Vomiting Heartburn Bleeding If allergic, bronchospasm and wheezing

What is the name of the condition when the patient forgets about the injured side after a stroke?

Neglect

Glascow Coma Scale (GCS)

Neurologic assessment of a patient's best verbal response, eye opening, and motor function.

Possession of __________ is an indication that the patient has a history of angina pectoris.

Nitroglycerin

You are on the scene of a 64-year-old male with substernal chest pain. His vital signs are blood pressure 90/70, pulse 80, respiratory rate 20, and oxygen saturation 95 percent on room air. He has a history of angina pectoris and high cholesterol. The patient is not allergic to any medications. What medications should you administer?

Oxygen and aspirin

What happens in the case of a mechanical pump failure?

Oxygen causes the death of a portion of the myocardium - the dead area can no longer contract and pump - Can lead to cardiac arrest, shock, pulmonary edema, or congestive heart failure

Angina pectoris

PAIN IN THE CHEST, occurring when the blood supply to the heart is reduced and a portion of the heart muscle is not receiving enough oxygen - Typically occurs after stress and exertion - Frequently pain will diminish after patient ceases whatever is causing stress and exertion

What can cause an increase in pulmonary vascular resistance?

PVR is increased by constriction, obstruction (e.g. emboli), or compression of the pulmonary vasculature or by backpressure from the left heart. Hypoxemia, acidosis, and release of histamine from an allergic reaction also cause vasoconstriction with an increase in PVR.

You are on-scene with a 48-year-old unresponsive male patient. Bystanders state he complained of chest pain and then suddenly collapsed. Vital signs are blood pressure 68/p, pulse 36, and respiratory rate 3. He is unresponsive to painful stimuli. After performing the primary assessment and treating all life threats, what is your next intervention?

Package the patient for rapid transport.

Angina Pectoris

Pain in the chest, which occurs when - the blood supply to the heart is reduced - a portion of the heart muscles is not receiving enough oxygen

What are signs and symptoms of Acute Coronary Syndrome?

Pain, pressure or discomfort in the chest or epigastrium Difficulty breathing Palpitations Sudden onset of sweating and nausea or vomiting Anxiety Generalized weakness Abnormal pulse Abnormal BP

Which of the following BEST describes a fluttering sensation in the chest?

Palpitations

autonomic nervous system

Part of the nervous system that governs involuntary operations, ie. HR, BP, breathing, digestion.

Simple or complex

Partial seizures have two classifications:

What are the contraindications for aspirin?

Patient cannot swallow Patient is allergic Patient had history of asthma Patient has gastrointestinal ulcer or recent bleeding Patient has a known bleeding disorder

What would be the result of a low oxygen level in the bloodstream?

Patient could be anxious, restless, and confused.

Which of the following terms refers to swelling of the lower extremities seen in many cardiac patients?

Pedal edema

Which of the following actions regarding AED operation is correct?

Perform CPR while the AED is being attached

Name Medications Which R Used 2 Control Epileptic Seizures

Phenytoin (Dilantin) Phenobarbital Carbamazepine (Tegretol)

dependent edema

Pitting edema of the lower legs is AKA

What steps do we take to correct ACS?

Place in position of comfort Administer oxygen if less than 94 pct or exhibiting symptoms of criticality Transport if: no history of cardiac problems, history of problems but no nitroglycerin, BP below 90 Give nitroglycerine if prescribed already and no contraindications Give aspirin if medical direction authorizes and no contraindications

If a patient with a pacemaker needs to be deliberated, the EMT should....

Place the pad several inches away from the pacemaker battery.

You respond to the scene of a 4-year-old female patient in cardiac arrest. CPR is in progress. What is the most likely cause of the cardiac arrest?

Pneumonia

Acute myocardial infarction

Portion of myocardial dies due to oxygen starvation

hemoglobin

Portion of the blood that transports the majority of oxygen.

A drug that increases the contractility of the heart is known as a positive?

Positive inotropic drugs >> increases the force and velocity of contraction and myocardial O2 consumption

Your cardiac arrest patient had a return of a spontaneous circulation. Medical direction is preparing to induce hypothermia. This treatment is part of:

Post cardiac arrest care

You have just arrived at an airport terminal where an airport employee has applied an AED to a patient who collapsed inside the gate area. The AED delivers the third shock as you place your equipment on the floor next to the patient. Which of the following should you do next?

Prepare the patient for transport

What is used to determine the afterload of the right ventricle?

Pulmonary Vascular Resistance (PVR)

What method of measuring cardiac output is most popular in ICU's around the country?

Pulmonary artery catheter (PAC) thermodilution method (TDCO >> thermodilution CO measurement)

Which of the following is the name given to the condition in which fluid accumulates in the lungs?

Pulmonary edema

left sided CHF

Pulmonary edema might indicate

Defibrillation is indicated for which of the following situations?

Pulseless ventricular tachycardia

The filling pressure of the right heart also is known as the _______ pressure.

RAP (right atrial pressure) CVP (central venous pressure)

What effect does an increase in intrathoracic pressure have on venous return?

Raised intrathoracic pressure decreases venous return

Ventricular tachycardia (V-Tach)

Rapid heartbeat -if rapid enough it will not allow the hearts chambers to fill with enough blood between beats to produce blood flow sufficient enough to meet the body's needs

What is true about ejection fraction?

Represents the percentage of the end-diastolic volume that is ejected with each beat. Normal EF is 65%-70% EF is either measured directly or calculated (SV/EDV) EF declines as cardiac function deteriorates When the EF falls to the 30% range, a patients exercise tolerance is severely limited because of the heart's inability to maintain an adequate CO

Pedal edema and JVD are signs of:

Right heart failure

Swollen ankles and coughing up pink sputum may cause you to suspect that the patient is experiencing...

Right heart failure

What is the beneficial action of a beta blocker​ medication?

SLOWS the heart rate

What does not have a direction effect on stroke volume?

SV is determined by three factors: Preload Afterload Contractility

FEBRILE SEIZURES

Seizures that result from sudden high fevers, particularly in children. Febrile seizures R typically well tolerated by children however rapid transport is still required.

Febrile seizures

Seizures that result from sudden high fevers; most often seen in children.

Which of the following BEST describes an appropriate shock sequence for the patient in pulseless VT?

Shock, 2 minutes of CPR, analyze, shock again

angina pectoris

Short lived chest discomfort caused by a temporary or partial blockage of blood flow to the myocardium. Pain in chest occurring when blood supply to heart is reduced.

dyspnea?

Shortness of breath; Difficulty breathing; Labored.

Dyspnea

Shortness of breath; labored or difficulty breathing

A sudden, severe headache, often described as the worst pain the patient has ever had.

Sign of a hemorrhagic stroke:

Bradycardia?

Slow heart rate. ( less than 60 bpm)

What does a beta blocker do?

Slows the heart and makes it beat less strongly to avoid future AMIs

hich of the following is the beneficial action of a beta blocker medication?

Slows the heart rate

Dysarthria

Slurred speech.

How do we administer aspirin?

Small chewable 81mg tablets

atria

Small heart chambers that contract to "top off" the ventricles.

If a patient that you suspect is having cardiac issues does not complain of difficulty breathing, what should you do?

Specifically ask them about it

What is most commonly described for chest pain?

Squeezing, dull,crushing, or heavy.

The strength of contraction of the myocardial muscle fibers is related directly to the amount of stretch on the fiber before contraction is a definition of:

Starling's Law

Uppers

Stimulants such as amphetamines.

What is the first thing you should do if a patient goes into (or back into) cardiac arrest during transport?

Stop the vehicle

SPECIAL STROKE CONSIDERATION FOR GERIATRICS

Stroke common in this age group overtime the brain gradually deteriorates and shrinks as a part of the normal aging process. This makes the patient more susceptible 2 injury, & lowers mental status.

LIFE THREATENING CAUSES OF HEADACHES

Stroke, tumor, infection of the CNS or hypertension. Headaches associated w other Sx or headaches described as the worst pain ever felt should B considered serious.

How do we administer nitroglycerin?

Sublingual tablets or spray

Antidote

Substance that will neutralize a drug

Common causes of Febrile seizures

Sudden high fever

Delirium tremens

Sweating, trembling, anxiety

Edma

Swelling from fluid

To enable the heart of a well-trained athlete to produce a cardiac output of up to 35 L/min, stimulation of the heart muscle is needed through what?

Sympathetic Nervous System

Possible Causes of Altered Mental Status

TIPS AEIOU T - Trauma, I - Infection, P - Psychogenic Causes, S - Seizure / Syncope A - Alcohol, E - Electrolytes, I - Insulin, 0 - Opiates, U - Uremia (Toxic condition resulting in kidney disease)

You are transporting a 50-year-old male patient whom you successfully defibrillated at the scene. You are 5 minutes away from the hospital when the patient goes back into cardiac arrest. Which of the following is the best course of action?

Tell the driver to stop, analyze the cardiac rhythm, and deliver a shock as necessary.

CAUSES OF TENSION HEADACHES

Tension headaches R most common & R caused by muscle contractions in the head & neck. They R attributed 2 stress. Squeezing, dull or aching do not require med. attention.

systole

Term used for a flat line on the EKG.

AV node

The "gatekeeper node" of the heart, slows conduction for a split second to allow preload of ventricles.

Which of the following statements about the AED is​ correct? A. The AED can always analyze the rhythm in a moving vehicle. B. AEDs can analyze in a moving vehicle but cannot shock. C. Biphasic AEDs can analyze the rhythm in a moving vehicle. D. The AED can encounter difficulty in analyzing a rhythm in a moving vehicle.

The AED can encounter difficulty in analyzing a rhythm in a moving vehicle.

Which of the following statements regarding the AED and defibrillation is correct?

The AED will not analyze the rhythm of a moving patient.

When using an AED, which of the following people is responsible for calling to "clear" the patient before delivering a shock?

The EMT operating the defibrillator

F - Facial droop (ask the patient to smile; look for a droop on either side of the patient's face) A - Arm drift (ask the patient to close his or her eyes and hold arms out with palms up, then watch for one to drift down) S - Speech (ask the patient to repeat a simple phrase, then listen for slurred speech) T - Time (note the time the patient was last seen acting normally)

The FAST mnemonic can also be used in assessing for a stroke:

automaticity

The ability of cardiac muscle cells to contract without stimulation from the nervous system.

carotids

The arteries that feed the skull

circumflex

The artery on the heart that goes around to the posterior surface.

The brain stem, the cerebellum, and the cerebrum.

The brain is divided into three major parts:

Oxygen, glucose, and temperature levels.

The brain is most sensitive to these changes:

What supply oxygen to the heart?

The coronary arteries

LAD

The coronary artery that feeds the left side of the heart.

Congestive Heart Failure (CHF)

The failure of the heart to pump efficiently, leading to excessive blood or fluids in the lungs, the body, or both A condition of excess fluid buildup in the lungs and/or other organs and body parts because of inadequate pumping of the heart - Is often a complication of an AMI - Often leads to edema

- Hypoglycemia - A postictal state (period following a seizure that lasts between 5 and 30 minutes, characterized by labored respirations and some degree of altered mental status) - Subdural or epidural bleeding (a collection of blood near the skull that presses on the brain)

The following conditions may appear to be a stroke:

- Facial drooping* - Sudden weakness or numbness in the face, arm, leg, or one side of the body - Decreased or absent movement and sensation on one side of the body - Lack of muscle coordination (ataxia) or loss of balance - Sudden vision loss in one eye; blurred or double vision - Difficulty swallowing (a primary reason for good airway management in a patient with a stroke) - Decreased level of responsiveness - Speech disorders - Aphasia; difficulty expressing thoughts or inability to use the right words (expressive aphasia) or difficulty understanding spoken words (receptive aphasia) - Slurred speech (dysarthria)* - Sudden and severe headache - Confusion - Dizziness - Weakness - Combativeness - Restlessness - Tongue deviation - Coma - Arm drift* - Grip

The general signs and symptoms of stroke include the following:

Asystole

The heart has ceased generating electrical impulses. Also known as flatline, is a state of no electrical activity from the heart and therefore no blood flow. It results in cardiac arrest. The heart has ceased generating electrical impulses "flatline"

What happens in v-fib?

The heart is basically "quivering" due to disorganized electrical activity. 50 percent of cases

What happens in v-tach?

The heart is beating too quickly to refill. 10 percent of cases

myocardium

The heart muscle

What happens in pulseless electrical activity?

The heart muscle has failed even though the electrical activity is normal. 15 to 20 percent of cases

When the myocardium requires more oxygen:

The heart rate decreases significantly. The arteries supplying the heart dilate.

Tri-cuspid valve

The heart valve between the right atrium and right ventricle.

Which of the following statements concerning heart attacks and cardiac arrest is NOT true?

The most common initial rhythm in sudden cardiac death is asystole.

Tension headaches, migraines, and sinus headaches.

The most common types of headaches:

Ventricular Fibrillation (VF)

The most serious cardiac rhythm disturbance

Which of the following is the best destination for a patient with signs and symptoms of AMI?

The nearest hospital with PCI capability

You are on the scene of a 68-year-old male patient complaining of severe chest pain for the last 20 minutes. He has a previous history of myocardial infarction and states it feels "just like the last time." You have applied oxygen and assisted him in administering aspirin and nitroglycerin with no reduction in the chest pain. Your nearest facility is 5 minutes away, a Level III Trauma Center is 10 minutes away, and a hospital with cardiac catheterization capabilities is 20 minutes away. The patient is requesting to be transported to his cardiologist's hospital, which is 30 minutes away. Which hospital is the best destination?

The nearest hospital with cardiac catheterization

What can affect cardiac output?

The normal CO for an individual varies with age, sex (10% higher in men), body size, blood viscosity (hematocrit), and the tissue demand for oxygen

Computed tomography (CT) scan of the head

The only reliable way to tell whether there is bleeding in the brain is with a special type of imaging test called...

Which of the following symptoms would lead to the EMT to believe that a patient's headache is caused by sinus congestion?

The pain is worse when bending over.

Which of the following BEST describes the pain associated with acute coronary​ syndrome? A. The pain is a pressure in the chest. B. The pain is only a mild discomfort. C. The pain will go away after a few hours. D. The pain may be chest​ pressure, mild​ discomfort, or not present at all.

The pain may be chest​ pressure, mild​ discomfort, or not present at all.

Your patient is a 49-year-old male with a history of heart problems. Assuming he has a bottle of nitroglycerin tablets with him, under which of the following circumstances could you administer the nitroglycerin to the patient?

The patient complains of chest pain and has a blood pressure of 132/90 mmHg.

Which of the following situations could contraindicate the administration of​ aspirin? A. The patient is having a heart attack. B. The patient is taking any other medications. C. There is no water available to take the pills with. D. The patient has a history of GI bleeding.

The patient has a history of GI bleeding.

In which of the following situations would the administration of aspirin to a cardiac patient be prohibited?

The patient has a history of asthma.

Complex partial seizure

The patient has an altered mental status and does not interact normally with his environment.

brachial

The pulse you feel inside the bicep is the

dorsalis pedis (pedal)

The pulse you feel on top of the foot or next to the medial ankle.

The greatest determinant of peripheral resistance to blood flow is?

The radius of the vessels

ANEURYSM

The swelling or enlargement of part of a blood vessel,resulting from weakening of the vessel wall.

fibrillation or tachycardia

The two cardiac rhythms that we can shock are ventricular

Ischemic and hemorrhagic.

The two main types of stroke:

What is the epigastrium?

The upper abdomen

pulmonary valve

The valve between the right ventricle and pulmonary artery.

When ventricular function curves are performed on a patient with a suspected failing right ventricle, how does cardiac output typically change as central venous pressure increases?

There is little and slow change in cardiac output for large changes in PCWP increases, but the curve then peaks (at approximately 5 L/min) and output begins to decline after PCWP reaches 20 mm Hg

Which of the following is an advantage of using a mechanical CPR device?

They provide a consistent rate and depth of compression

HTN

This may cause hemorrhagic strokes.

Migraine headaches

Thought to be caused by changes in blood vessel size in the base of the brain. Pain is usually described as pounding, throbbing, or pulsating. Often associated with nausea and vomiting and may be preceded by visual warning signs such as flashing lights or partial vision loss.

What is the max allowable number of doses for nitroglycerin?

Three. Administer first dose, wait five minutes and administer next if symptoms continue.

Cincinnati Prehospital Stroke Scale

To rapidly identify stroke 1) Facial droop 2) Arm drift 3) Speech "The sky is blue in Cincinnati."

You respond to a residence for a child who is having a seizure. Upon arrival at the scene, you enter the residence and find the mother holding her child, a 2-year-old male. The child is conscious and crying. According to the mother, the child has been running a high fever and then experienced a seizure that lasted approximately 3 minutes. You should:

Transport the child to the hospital and reassure the mother en route.

Which of the following is the​ EMT's role in caring for a patient with chest​ pain?

Treat the patient as though he were having a heart attack

Which of the following is the EMT's role in caring for a patient with chest pain?

Treat the patient as though he were having a heart attack.

The average cardiac output for adult men and women at rest is 5 L/min?

True. Normal range is 4 to 8 L/min

Common causes of Structural seizures

Tumor (benign or cancerous) Infection (brain abscess) Scar tissue from injury Head trauma Stroke

Generalized and partial (focal)

Two basic groups of seizures:

Volatile chemicals

Vaporizing compounds such as cleaning supply's

A paramedic unit has arrived on the scene of a cardiac arrest where you have been performing CPR and using an AED. The crew has started an IV and inserted an endotracheal tube. How will this change how you do your​ job? A. You no longer need to do compressions because the paramedics have their monitor. B. Ventilations may be delivered asynchronously from chest compressions now. C. The AED should be immediately removed. D. You will now shock once every minute because drugs may be given.

Ventilations may be delivered asynchronously from chest compressions now. "asynchronous" ventilations during CPR in patients with an advanced airway, that is, breaths interposed every six seconds without interruption in CPR—OR 10 breaths/minute without stopping CPR.

You should NOT perform CPR on a patient with which type of cardiac implant device (unless directed to do so by medical direction)?

Ventricular assist device (VAD)

Which electrical disturbance of the heart would look like "a quivering bag of worms"?

Ventricular fibrillation

Which of the following cardiac dysrhythmias cannot produce a pulse?

Ventricular fibrillation

What are AED shockable rhythms?

Ventricular fibrillation Ventricular tachycardia

What are the different types of pulseless activity?

Ventricular fibrillation Ventricular tachycardia Pulseless electrical activity Asystole

What is the primary electrical disturbance resulting in cardiac arrest?

Ventricular fibrillation in 50 percent of cases

In which of the following dysrhythmias would it be most important to apply and engage an automatic external​ defibrillator? A. Asystole B. Ventricular tachycardia C. Atrial tachycardia D. Pulseless electrical activity

Ventricular tachycardia

Hemiparesis

Weakness on one side of the body.

A patient with altered mental status cannot reliably tell you what is wrong, and there may be more than one cause.

What is the most significant difference between a patient with altered mental status and other emergencies?

Subarachnoid hemorrhages

When a berry aneurysm is overstretched and ruptures, blood spurts into an area between two of the coverings of the brain called the subarachnoid space.

Withdrawal

When body is deprived of substance

dissecting aneurysm

When the layers of a blood vessel begin to separate, allowing blood to flow between the layers.

Can we use the AED on a patient with a pacemaker?

Yes, but place it in a location it will not touch the pacemaker

Rapid transport

You have checked ABCDs on your Pt. He is pale, cool , profusely diaphoretic, presents Levine's sign, and complains of crushing CP. He is bradycardic and hypotensive, you apply oxygen, admin. aspirin and nitro. What is the next MOST IMPORTANT treatment for this Pt. ?

Acute Coronary Syndrome (ACS)

a blanket term used to represent any symptoms related to lack of oxygen (ischemia) in the heart muscle; also called "cardiac compromise"

Common signs and symptoms of a hypertensive emergency include:

a bounding pulse, a severe headache, and dizziness.

Thrombus

a clot formed of blood and plaque attached to the inner wall of an artery or vein

Asystole

a condition in which the heart has ceased generating electrical impulses

Ventricular Fibrillation (V-fib)

a condition in which the heart's electrical impulses are disorganized, preventing the heart muscle from contracting normally

Ventricular Tachycardia (V-tach)

a condition in which the heartbeat is rapid; if rapid enough, ventricular tachycardia will not allow the heart's chambers to fill with enough blood between beats to produce blood flow sufficient to meet the body's needs

Acute coronary syndrome (ACS) is a term used to describe:

a group of symptoms that are caused by myocardial ischemia.

Which of the following is NOT a common sign or symptom associated with malfunction of an implanted cardiac pacemaker?

a rapid heart rate

Phases Of A General Tonic-Clonic Seizure

a. Aura - since of deja vu. b. Tonic - lasting a few seconds, person loses consciousness & skeletal muscles tense. c. Clonic - muscles contract & relax rapidly. d. Postictal - state of altered consciousness person enters after experiencing a seizure.

Congenital?

abnormality in the heart that develops before birth.

Pulmonary Edema

accumulation of fluid in the lungs

Cardiopulmonary Resuscitation (CPR)

actions take to revive a person by keeping the person's heart and lungs working

A 65-year-old female complains of upper abdominal pain, nausea, and weakness. She tells you she had a previous heart attack two years ago. She is slightly sweaty, and her vital signs are P 88, R 20, BP 89/60. Her pulse oximeter reads 95 percent on room air. You should first: A administer high-concentration oxygen. B administer aspirin. C administer nitroglycerin. D complete a detailed physical examination.

administer aspirin. her BP is too low to administer Nitroglycerin

A 67-year-old male complains of chest pressure. He is gray and diaphoretic. His vital signs are P 94, R 24, BP 190/90, and his pulse oximetry reads 91 percent. You should first: A administer oxygen. B administer aspirin. C administer nitroglycerin. D complete a detailed physical examination.

administer oxygen

A 66-year-old female with a history of hypertension and diabetes presents with substernal chest pressure of 2 hours' duration. Her blood pressure is 140/90 mm Hg, her pulse is 100 beats/min and irregular, and her respirations are 22 breaths/min with adequate depth. The patient does not have prescribed nitroglycerin, but her husband does. You should

administer oxygen, give her 2 (81mg each tablet) chewable aspirin, and assess her further.

As you arrive for a patient in cardiac​ arrest, bystanders are analyzing the patient with an AED. You​ should: A. remove the AED and apply your own. B. allow the AED to shock before you take over care. C. turn off the AED and check for a pulse. D. stay back until bystanders transfer care to you.

allow the AED to shock before you take over care.

When assessing the​ patient's symptoms, you​ should: A. use​ closed-ended questions for efficiency. B. give the patient a list of specific symptoms to confirm or deny. C. let the patient know you are only interested in the severity of pain. D. allow the patient to describe symptoms in his own words.

allow the patient to describe symptoms in his own words.

where does the pain usually radiate too?

along arms, down upper abdomen, and up to the jaw bone.

Ischemia

an inadequate blood supply to an organ or part of the body, especially the heart muscles.

Your patient has a weakened section of the arterial wall that is ballooning. This is known as​ a(n): A. occlusion. B. embolism. C. thrombus. D. aneurysm.

aneurysm

Which of the following refers to chest pain that occurs when blood supply to the heart is reduced and a portion of the heart muscle is not receiving enough​ oxygen? A. Acute myocardial infarction B. Aneurysm C. Angina D. Hypertension

angina

The MOST common reason that many people experiencing AMI do not seek immediate medical attention is because they

are in denial

Which of the following medications is commonly given to patients with chest pain to prevent blood clots from forming or getting bigger?

asprin

A 49-year-old male presents with an acute onset of crushing chest pain and diaphoresis. You should:

assess the adequacy of his respirations.

Upon arriving at the residence of a patient with a possible cardiac problem, it is MOST important to:

assess the scene for potential hazards.

You are dispatched to a residence for a 56-year-old male with an altered mental status. Upon arrival at the scene, the patient's wife tells you that he complained of chest pain the day before, but would not allow her to call EMS. The patient is semiconscious; has rapid, shallow respirations; and has a thready pulse. You should:

begin ventilatory assistance.

Which of the following types of defibrillators will adjust the amount of energy used on the basis of​ impedance? A. Triphasic defibrillator B. Monophasic AED C. Biphasic AED D. Implanted defibrillator

biphasic AED

Acute Coronary Syndrome (ACS)?

blanket term for lack of oxygen in heart muscle.

Embolism

blockage of a vessel by a clot or foreign material brought to the site by the blood current

embolism?

blockage of a vessel by a clot or foreign object.

Which term refers to a heart rate of below 60 beats per​ minute? A. Agonal B. Tachycardia C. Bradycardia D. Dysrhythmia

bradycardia

The inferior vena cava returns deoxygenated blood to the right side of the heart from all of the following areas, EXCEPT the:

brain

The head and brain receive their supply of oxygenated blood from the:

carotid arteries.

Pulmonary embolism / signs and symptoms

caused by a blockage in the pulmonary artery chest pain, pedal edema in one leg Risk factors include long sedentary periods

Left side heart failure

causes fluid backup in the lungs typically presents with pulmonary edema and respiratory distress

Right side heart failure

causes fluid backup within the body presents with pedal edema and JVD

Interruption of cerebral blood flow may result from all of the following, EXCEPT:

cerebral vasodilation.

Which of the following is an expected side effect of​ aspirin? A. Fever B. Increased clotting C. Heartburn D. Hypertension

heartburn

A 66-year-old female has collapsed while in line at the post office. She is unresponsive and apparently not breathing. You should first: A open the patient's airway. B deliver two rescue breaths. C insert an oropharyngeal airway. D check for a pulse

check for a pulse

Your patient was pulseless but had a return of spontaneous pulses after one shock from the AED. Before​ transport, you notice that the patient is again unresponsive. You should​ NEXT: A. continue ventilations. B. check the pulse. C. assess the blood pressure. D. transport the patient.

check the pulse.

When would it be MOST appropriate for a patient to take his or her prescribed nitroglycerin?

chest pain that does not immediately subside with rest

An​ 80-year-old male is presenting with pedal​ edema, JVD, and dyspnea. You should​ suspect: A. anaphylaxis. B. congestive heart failure. C. pulmonary embolism. D. ​multi-system trauma.

congestive heart failure

Rapid, labored breathing in a patient with signs and symptoms of AMI should make you suspicious for:

congestive heart failure.

Which of the following is NOT a function of the sympathetic nervous system?

constriction of blood vessels in the muscles

You are treating your cardiac arrest patient when a​ "No Shock​ Indicated" message appears. You should​ NEXT: A. continue CPR for two minutes. B. insert an oral airway. C. reanalyze. D. ventilate 20 times per minute

continue for two minutes

The myocardium receives oxygenated blood from the __________, which originate(s) from the __________.

coronary arteries, aorta

Conditions that narrow or block the arteries of the heart are​ called: A. pulmonary embolism. B. coronary artery disease. C. cardiac dysrhythmia. D. pulmonary edema.

coronary artery disease

What kind of chest pain is often associated with cardiac problems?

crushing, dull, heavy, squeezing; commonly radiates along the arms, down to the upper abdomen, or up to the jaw

You must reassess your patient within two minutes after administering​ nitroglycerin, as one of the side effects​ is: A. numbness and tingling in the extremities. B. a decrease in blood pressure. C. a sudden decrease in heart rate. D. a brief feeling of shortness of breath.

decrease in blood pressure

Cardiogenic shock following AMI is caused by:

decreased pumping force of the heart muscle

ALS personnel have just placed an advanced airway in an adult cardiac arrest patient. You​ should: A. ventilate the patient twice after every 15 compressions. B. deliver 2 breaths after every 30 compressions. C. deliver asynchronous ventilations at 8 to 10 breaths per minute. D. ventilate the patient at 12 to 20 breaths per minute.

deliver asynchronous ventilations at 8 to 10 breaths per minute. "asynchronous" ventilations during CPR in patients with an advanced airway, that is, breaths interposed every six seconds without interruption in CPR—OR 10 breaths/minute without stopping CPR.

Defibrillation

delivery of electrical shock to stop the fibrillation of heart muscles and restore a normal heart rhythm

Which of the following signs is commonly observed in patients with right-sided heart failure?

dependent edema

A percutaneous transluminal coronary angioplasty (PTCA) restores blood flow to the ischemic myocardium by:

dilating the affected coronary artery with a small inflatable balloon.

Nitroglycerin relieves cardiac-related chest pain by

dilating the coronary arteries and improving cardiac blood flow. reducing the overall workload on the heart through vasodilation and reduced blood pressure.

Signs and symptoms of a hypertensive emergency would MOST likely be delayed in patients who:

have chronic hypertension.

After administration of nitroglycerin to a patient with chest​ pain, which is a side effect you MOST might expect to​ see? A. The patient develops hypertension. B. The patient becomes dehydrated. C. The patient becomes irritable. D. The patient develops a headache.

headache

Coronary Artery Disease (CAD)

diseases that affect the arteries of the heart (often a result of the buildup of fatty deposits on the inner walls of the coronary arteries)

A 66-year-old woman presents with a stabbing pain in the middle of her chest that radiates to her back. She tells you that the pain suddenly began about 30 minutes ago and has been severe since the onset. She has a history of hypertension, but admits to being noncompliant with her antihypertensive medications. When you assess her, you find that her blood pressure is significantly higher in her left arm than it is in her right arm. What are her signs and symptoms MOST indicative of?

dissecting aortic aneurysm

dysrhythmia?

disturbance in heart rate or rhythm

One of the side effects of administering nitroglycerin to a cardiac patient could​ involve: A. apnea. B. hypertension. C. a drop in blood pressure. D. chest pain.

drop in blood pressure

Prior to attaching the AED to a cardiac arrest patient, the EMT should:

dry the chest off if it is wet.

Crushing chest pain may also be described as...

dull, squeezing, heavy

A patient whose speech is slurred and difficult to understand is experiencing:

dysarthria.

What of the following is the single MOST important factor in determining survival from cardiac​ arrest? A. Immediate recognition B. Effective ALS C. Early CPR D. Rapid transport

early CPR

In the cardiac conductive​ system, what cause the heart to contract are specialized muscles​ and: A. the major blood vessels. B. blood pressure. C. electrical impulses. D. circulation between the heart and the lungs

electrical impulses

For both the Lucas and​ Auto-Pulse CPR​ devices, you would take Standard Precautions​ and: A. close the Lifeband over the​ patient's chest. B. position the pressure pad so that it touches the chest. C. ensure CPR is in progress and effective. D. apply the stabilization strap.

ensure CPR is in progress and effective.

Prior to defibrillating a patient with an AED, it is MOST important that you:

ensure that no one is touching the patient.

The main legal risk in using the AED is:

failing to deliver a shock when one is needed.

The MOST common error associated with the use of the AED is

failure of the EMT to ensure the battery is charged

Which of the following is of LEAST pertinence when obtaining medical history information from a patient complaining of chest discomfort?

family history of hypertension

Which of the following descriptors is MOST common in a patient who is experiencing chest pain due to a problem with the​ heart? A. ​"Tearing" B. ​"Ripping" C. ​"Heavy" D. ​"Burning"

heavy

Common side effects of nitroglycerin include all of the following, EXCEPT

hypertension.

Common side effects of nitroglycerin include all of the following, EXCEPT:

hypertension.

The most significant risk factor for a hemorrhagic stroke is:

hypertension.

what is a metabolic cause for a seizure?

hypoglycemia

Major risk factors for AMI include all of the following, EXCEPT:

hypoglycemia.

Which of the following signs or symptoms would you NOT expect to encounter in a patient with congestive heart failure?

hypotension and flat jugular veins

After the AED has delivered a shock, the EMT should:

immediately resume CPR.

Which of the following would cause the greatest increase in cardiac output?

increased heart rate and increased stroke volume

Agonal Breathing

irregular, gasping breaths that priced apnea and death

When afterload increases:

it becomes harder for the ventricle to push blood through the blood vessels.

The AED is MOST advantageous to the EMT because:

it delivers prompt defibrillation to patients with ventricular fibrillation.

When using a mechanical CPR​ device, you​ should: A. switch to manual CPR for every other set. B. stop ventilations during transport. C. limit interruptions in chest compressions to 10 seconds or less. D. limit use of the device to 10 minutes or less.

limit interruptions in chest compressions to 10 seconds or less.

You suspect that your patient is experiencing acute coronary syndrome. Your hospital destination should be determined​ by: A. traffic conditions. B. local protocols. C. patient preference. D. the​ patient's insurance.

local protocols

A patient with atherosclerotic heart disease experiences chest pain during exertion because the:

lumen of the coronary artery is narrowed and cannot accommodate increased blood flow.

The EMT should use an AED on a child older than 1 year if:

pediatric pads and an energy-reducing device are available.

Your EMS team is performing CPR on a 60-year-old male in cardiac arrest. You connect the AED, push the analyze button, and receive a "no shock advised" message. You should:

perform CPR for 2 minutes and reassess

Angina pectoris most commonly occurs when:

myocardial oxygen demand exceeds the supply

In contrast to an Acute Myocardial Infarct (AMI), angina pectoris most commonly occurs when:

myocardial oxygen demand exceeds the supply

Angina pectoris occurs when:

myocardial oxygen demand exceeds the supply.

An acute myocardial infarction (AMI) occurs when:

myocardial tissue dies secondary to an absence of oxygen.

According to the NM EMT-Basic Scope of Practice, which of the following MUST be in place prior to the administration of Nitro to a patient complaining of chest pain?

obtain authorization from medical control

Prior to assisstin a patient with his or her prescribed nitroglycerin, the EMT should:

obtain authorization from medical control

Prior to assisting a patient with his or her prescribed nitroglycerin, the EMT must:

obtain authorization from medical control.

In contrast to AMI, a dissecting aortic aneurysm:

often presents with pain that is maximal from the onset.

You are treating a patient with signs and symptoms of a myocardial infarction​ (MI). What is the most important drug you should​ administer?

oxygen

Angina Pectoris

pain in the chest occurring when blood supply to the heart is reduced and a portion of the heart muscle is not reaching enough oxygen

You and your partner arrive at the scene of a middle-aged man who collapsed about 5 minutes ago. He is unresponsive, apneic, and pulseless. Bystanders are present, but have not provided any care. You should:

perform five cycles of high-quality CPR and then apply the AED.

A 67-year-old female presents with difficulty breathing and chest discomfort that awakened her from her sleep. She states that she has congestive heart failure, has had two previous heart attacks, and has prescribed nitroglycerin. She is conscious and alert with adequate breathing. Her blood pressure is 94/64 mm Hg and her heart rate is 120 beats/min. Treatment for this patient includes:

placing her in an upright position.

In the mnemonic​ OPQRST, the​ "P" stands​ for: A. pallor. B. pressure. C. provocation. D. perspiration.

provocation

Which of the following blood vessels transports oxygenated blood?

pulmonary veins

The AED has delivered a shock to an elderly male in cardiac arrest. Following 2 minutes of CPR, you reanalyze the patient's cardiac rhythm and receive a "no shock advised" message. After further resuscitation, you restore a palpable carotid pulse. Your next action should be to:

reassess airway and breathing and treat accordingly

After assisting your patient with his or her nitroglycerin, you should

reassess the blood pressure within 5 minutes to detect hypotension.

After assisting your patient with his or her nitroglycerin, you should:

reassess the blood pressure within 5 minutes to detect hypotension.

Which of the following would be considered a contraindication to the administration of aspirin in acute coronary syndrome? A Recent gastrointestinal bleeding B Hypertension C Hypotension D Nausea

recent gastrointestinal bleeding

A study into the effects of disease on blood composition would focus​ on: A. red and white blood​ cells, platelets, and plasma. B. systolic and diastolic pressures. C. the peripheral and central pulses. D. the​ arteries, veins,​ arterioles, venules, and capillaries.

red and white blood​ cells, platelets, and plasma.

Which of the following is NOT one of the links in the chain of​ survival? A. Rapid defibrillation B. Immediate recognition and activation C. Early CPR D. Rehabilitation

rehabilitation

You are assessing a 49-year-old man who complains of chest pressure that began the night before. He is conscious, but anxious, and tells you he has a history of angina and hypertension. After applying high-flow oxygen, you expose his chest to auscultate his lungs and note that he has a nitroglycerin patch on his right upper chest. His skin is cool and pale, his blood pressure is 78/50 mm Hg, and his pulse is 110 beats/min and irregular. You should:

remove the nitroglycerin patch, place him in a supine position and elevate his lower extremities, and prepare for immediate transport.

In contrast to monophasic defibrillation, biphasic defibrillation:

requires a lower energy setting.

Cardiovascular System

the heart and blood vessels

You are attempting to resuscitate a 59-year-old male in cardiac arrest. The AED has been attached and has finished analyzing. It states, "No Shock Advised." You should next: A reanalyze. B give two rescue breaths. C insert an oropharyngeal airway. D resume chest compressions.

resume chest compressions

You have just used an AED to deliver a shock to your cardiac arrest patient. You should​ immediately: A. begin artificial ventilations. B. ​re-analyze with the AED. C. check a carotid pulse. D. resume chest compressions.

resume chest compressions

Deoxygenated blood from the body returns to the:

right atrium.

Defibrillator pads are placed on the patient's chest with one pad to the

right of the upper sternum and the other pad just to the left and below the left nipple.

The right coronary artery supplies blood to the:

right ventricle and inferior wall of the left ventricle.

Which of the following veins is located inferior to the trunk?

saphenous

Hypoperfusion refers​ to: A. shock. B. edema. C. hypertension. D. dyspnea.

shock

when an electrical impulse reaches the AV node, it is slowed for a brief period of time so that :

sinoatrial (SA) node can reset and generate another impulse

The electrical impulse generated by the heart originates in the:

sinoatrial (SA) node.

In contrast to the sympathetic nervous system, the parasympathetic nervous system

slows the heart and respiratory rates.

In contrast to the sympathetic nervous system, the parasympathetic nervous system:

slows the heart and respiratory rates.

A​ 54-year-old male is found pulseless and apneic on the sofa. The scene is safe. You should​ first: A. start CPR and retrieve an AED. B. apply oxygen. C. call medical control. D. call for ALS.

start CPR and retrieve an AED.

A 67-year-old female with severe chest pain becomes unresponsive, pulseless, and apneic during transport. You should:

stop the ambulance, begin CPR, and attach the AED as soon as possible immediately begin CPR, apply an AED, stop along the side of the road to allow the AED to analyze, and shock as indicated

The purpose of defibrillation is to

stop the chaotic, disorganized contraction of the cardiac cells.

The purpose of defibrillation is to:

stop the chaotic, disorganized contraction of the cardiac cells.

The Lucas CPR device​ involves: A. removing the stabilization strap if the patient must be moved. B. providing​ bag-mask ventilation at a rate of two ventilations for every 30 compressions. C. stopping CPR just long enough to put a base plate under the patient. D. closing the Lifeband chest band over the​ patient's chest.

stopping CPR just long enough to put a base plate under the patient.

Hypertensive

systolic > 140 OR diastolic > 90

When the myocardium requires more oxygen

the arteries supplying the heart dilate.

When treating a patient with chest pain, you should assume that he or she is having an AMI because:

the cause of the pain cannot be diagnosed in the field.

Acute Myocardial Infarction (AMI)

the condition in which a portion of the myocardium dies because of oxygen starvation; often called a "heart attack" by laypersons - brought on by the occlusion of coronary arteries

Aneurysm

the dilation, or ballooning, of a weak ended section of the wall of an artery

Congestive Heart Failure (CHF)

the failure of the heart to pump efficiently, leading to excessive blood or fluids in the lungs, the body, or both

Pulseless Electrical Activity (PEA)

the heart muscle itself fails even though the electrical rhythm remains relatively normal - Means that the patient's heart muscle is severely and almost always terminally sick - Could mean that the patient has lost too much blood (maybe the heart COULD pump but there is no fluid to pump) A condition in which the heart's electrical rhythm remains relatively normal, yet the mechanical pumping activity fails to follow the electrical activity, causing cardiac arrest

Ventricular Fibrillation

the heart's electrical impulses are disorganized, preventing the heart from contracting normally

A dissecting aortic aneurysm occurs when:

the inner layers of the aorta become separated.

Ventricular tachycardia causes hypotension because:

the left ventricle does not adequately fill with blood

A​ 65-year-old male complains of chest pain that started about 45 minutes ago and is radiating up to his jaw. He feels nauseous and has very little energy. His vitals are respirations of​ 24, pulse of​ 110, and BP of​ 88/60. What would make you decide to transport this​ patient? A. The elevated pulse B. The lack of energy C. The low blood pressure reading D. The nausea

the low blood pressure reading

The key difference between angina and a myocardial infarction is​ that: A. angina typically occurs at rest. B. myocardial infarction is usually caused by stress. C. the pain of angina usually goes away with rest. D. the pain of myocardial infarction usually goes away on its own.

the pain of angina usually goes away with rest.

Which of the following is a contraindication for use of the​ AED? A. The patient has been down for over 10 minutes. B. The patient is under 8 years of age. C. The patient is hypothermic. D. The patient is in contact with metal.

the patient is in contact with metal

Prompt transport of a patient with a suspected AMI is important because:

the patient may be eligible to receive thrombolytic therapy.

Cardiac output may decrease if the heart beats too rapidly because

there is not enough time in between contractions for the heart to refill completely.

Cardiac output may decrease if the heart beats too rapidly because:

there is not enough time in between contractions for the heart to refill completely.

Most patients are instructed by their physician to take up to _______ doses of nitroglycerin before calling EMS

three

​Auto-Pulse is​ designed: A. to be started manually. B. to be started automatically. C. to start only if an advanced airway is in place. D. to start only if​ bag-valve mask ventilation is applied.

to be started manually

Considering high-performance CPR, what is the minimum depth of compression for an adult? A One inch B Two inches C Three inches D Half an inch

two inches

When documenting a patient's description of his or her chest pain or discomfort, the EMT should:

use the patient's own words.

Sudden death following AMI is MOST often caused by:

ventricular fibrillation.

Which of the following cardiac arrhythmias has the greatest chance of deteriorating into a pulseless rhythm?

ventricular tachycardia

Which of the following cardiac dysrhythmias would be shocked by an automated external defibrillator? A Ventricular tachycardia B Asystole C Pulseless electrical activity D Sinus tachycardia

ventricular tachycardia

tachycardia?

when heart rate is fast. (more than 100 bpm)

Expect to perform​ one-rescuer CPR: A. under direct ALS supervision. B. while en route to a medical facility. C. in any emergency response situation. D. when lifting and moving patients.

while en route to a medical facility.

Nitroglycerin is contraindicated in patients

who have experienced a head injury.

Contraindications for the administration of nitroglycerin include a​ patient: A. who is 22 years of age. B. who has already taken one dose of nitroglycerin without relief. C. who took Viagra in the last 48 hours. D. who is suffering from hypertension.

who took Viagra in the last 48 hours.

dilation

widening of the blood vessel is referred to as

A​ "pit crew" approach is involved​ in: A. ​high-performance CPR. B. emergency medical response. C. rapid defibrillation. D. advanced life support.

​high-performance CPR.

The 5 elements in the chain of survival

• Immediate recognition of cardiac arrest and activation of the emergency response system • Early CPR • Rapid defibrillation • Effective advanced life support • Integrated post-cardiac arrest care


Related study sets

Chapter 11 - Human Resource Management: Finding and Keeping the Best employees

View Set

Math 146-Wurts/ Chapter 6-Putting statistics to work

View Set

GERD and Other Esophageal Probs: NCLEX Qs on Chapter 54: IGGY

View Set

HW17: Homework - Ch. 17: Public Goods and Common Resources

View Set